You are on page 1of 117

Arkansas Tech University

MATH 2934: Calculus III


Dr. Marcel B. Finan

10.1 Three-Dimensional Coordinate Systems


In this section, we learn the aspects of the three-dimensional coordinate
system. We define the three dimensional coordinate system as shown
in Figure 10.1.1(a).

Figure 10.1.1
There are three coordinate axes: The xaxis, the yaxis, and the zaxis.
They are oriented as shown in Figure 10.1.1(a). The point O is called the
origin of the system. As shown in Figure 10.1.1(b), the coordinate system
determine three coordinate planes and these planes divide the space into
eight parts, each called an octant: Four octants above the xyplane and
four octants below the xyplane. The octants are numbered as shown in
Figure 10.1.2.

Figure 10.1.2
The first octant is determined by the positive x, y, and zaxes.
In the two-dimensional system two numbers are required to determine a

1
point. Likewise, in the three-dimensional coordinate system, a point in
space P requires three numbers: a, b, and c. More precisely, a point P in
space is uniquely determined by an ordered triple (a, b, c) as shown in Figure
10.1.3.

Figure 10.1.3

The numbers a, b, and c are the coordinates of P : a is the xcoordinate,


b is the ycoordinate and c is the zcoordinate. In terms of sets, the
three-dimensional coordinate system will be denoted by the set

IR3 = {(x, y, z) : x, y, z IR}.

Now, from the point P (a, b, c) we can create a box with each vertex having
the coordinates shown in Figure 10.1.4. The point Q(a, b, 0) is called the
orthogonal projection of P onto the xyplane. Likewise, R(0, b, c) is the
orthogonal projection of P onto the zyplane and S(a, 0, c) is the orthogonal
projection of P onto the xzplane.

Figure 10.1.4

The solutions to an equation of the form f (x, y, z) = 0 are represented by a


surface in IR3 .

2
Example 10.1.1
Sketch and describe each of the following surfaces in IR3 : (a) y = 3 (b)
y = x.

Solution.
(a) The surface is a plane parallel to the xz plane and located 3 units to
the right of it as shown in Figure 10.1.5(a).
(b) The surface is a plane that contains the zaxis, and intersects the
xyplane at the line y = x as shown in Figure 10.1.5(b)

Figure 10.1.5

The Distance Formula


The distance between two points P1 (x1 , y1 , z1 ) and P2 (x2 , y2 , z2 ) is given by
the distance formula
p
||P1 P2 || = (x2 x1 )2 + (y2 y1 )2 + (z2 z1 )2 .

This formula is easily derived from Figure 10.1.6. Indeed, using the Pythagorean
formula in the highlighted right triangle, we find

||P1 P2 ||2 = (x2 x1 )2 + (y2 y1 )2 + (z2 z1 )2 .

3
Figure 10.1.6

Example 10.1.2
Find the distance between the two points C(a, b, c) and P (x, y, z).

Solution. p
Using the distance formula, we find ||CP || = (x a)2 + (y b)2 + (z c)2

Example 10.1.3
Describe and sketech the surface (x a)2 + (y b)2 + (z c)2 = r2 , where
r > 0.

Solution.
The collection of all points in space whose distances from a fixed point
C(a, b, c) is equal to a positive number r is called a sphere. We call the
point C the center and the number r the radius of the sphere. This
definition, implies that ||CP || = r or equivalently

(x a)2 + (y b)2 + (z c)2 = r2 .

This last equation is called the standard form of the equation of a sphere.
The surface is shown in Figure 10.1.7

Figure 10.1.7

4
Example 10.1.4
Find the center and the radius of the sphere: 2x2 +2y 2 +2z 2 +6x+4y2z = 1.

Solution.
Using the method of completing the square, we find

2x2 + 2y 2 + 2z 2 + 6x + 4y 2z =1
   
2 9 2 2 1 9 1
2 x + 3x + + 2(y + 2y + 1) + 2 z z + =1 + + 2 +
4 4 2 2
 2  2
3 1
2 x+ + 2(y + 1)2 + 2 z =8
2 2
3 2 1 2
   
2
x+ + (y + 1) + z =4.
2 2

Hence, the center is C 23 , 1, 12 and the radius is 2




Example 10.1.5
Describe the region in IR3 represented by the compound inequality 1
x2 + y 2 + z 2 4, z 0.

Solution. p
An equivalent form of the given inequality is 1 x2 + y 2 + z 2 2. This
inequality represents the points in IR3 whose distance from the origin is at
least 1 and at most 2. Since z 0, we consider only those points that lie on
or below the xyplane. Thus, the inequality represents those points that
are between (or on) the sphere x2 + y 2 + z 2 = 1 and x2 + y 2 + z 2 = 4 and
beneath (or on) the xyplane as shown in Figure 10.1.8

Figure 10.1.8

5
Arkansas Tech University
MATH 2934: Calculus III
Dr. Marcel B Finan

10.2 Introduction to Vectors


In the previous calculus classes we have seen that the study of motion involved
the introduction of a variety of quantities which are used to describe the phys-
ical world. Examples of these quantities include distance, displacement, speed,
velocity, acceleration, force and mass. Some of these are characterized by a
single number and others require a number and a direction.
In general, objects in the physical world are divided into two categories: scalars
and vectors.
Scalars are objects that can be modeled or characterized using a single number.
Examples of scalars in the study of motion are distance, speed, and mass.
Vectors are physical objects or quantities that require both a distance and
a direction for their specification. In the study of motion, the quantities dis-
placement, velocity, acceleration, and force are examples of vectors.
The purpose of this section is to understand some fundamentals about vectors.

Notation and Terminology


By a displacement vector from a point A to a point B we mean an arrow
with its tail at A and its tip at B as shown in Figure 10.2.1.

Figure 10.2.1
The direction of the vector is the direction of the arrow. The distance between
A and B is known as the magnitude or length of the vector. We will rep-

resent this vector by AB and its magnitude by ||AB||. In many instances, the
endpoints of a vector are ignored and the vector is just denoted by ~v . In this
case, the magnitude of the vector will be denoted by ||~v ||.
We define the zero vector to be a displacement vector with zero length. We
will denote it by ~0. The zero vector has no direction.

Algebra of Vectors: Addition and Difference of Two Vectors


Starting from the bookstore, suppose that along a certain direction one can
reach a classroom after traveling a distance of 500 ft and then 300 ft in another
direction to reach the pool. By drawing the two displacement vectors one can
draw a third vector representing the walk from the bookstore straight to the
pool as shown in Figure 10.2.2.

1
Figure 10.2.2
If we let ~v and w
~ represent the vectors from the bookstore to the classroom and
the classroom to the pool, respectively, then the vector from the bookstore to
the pool is just the sum ~v + w ~ as shown in Figure 10.2.3. This clearly makes
sense, because a person can get to the pool from the bookstore by walking there
directly or by first walking to the classroom and then to the pool. Either way
he or she eventually reaches the pool.

Figure 10.2.3
Now, suppose the displacement vectors ~u and ~v from the bookstore to both the
classroom and the pool are known. What is the displacement vector ~x from
the classroom to the pool? Since ~u + ~x = ~v , we define ~x to be the difference
~x = ~u ~v . See Figure 10.2.4.

Figure 10.2.4
Multiplication of a Vector by a Scalar
If a vector ~u and a scalar are given we would like to know what does ~u
stand for? The result is a vector of magnitude ||~u|| = ||||~u|| and in the same
direction of ~u if > 0 and opposite direction if < 0. Figure 10.2.5 shows three
vectors, ~u, 12 ~u, and 2~u.

2
Figure 10.2.5
For example, the difference ~u ~v is just the sum ~u + (1)~v as shown in Figure
10.2.6.

Figure 10.2.6
Parallel Vectors
The operation of multiplying a vector by a scalar leads to the following defi-
nition: We say that two vectors ~u and ~v are parallel if and only if ~u = ~v
for some non-zero scalar . When = 1, i.e., ~u = ~v , we say that ~u and ~v are
equivalent.

The Components of Vectors


Consider the three dimensional Cartesian coordinate system. We associate a
direction with each of the three axes of this system. In particular we define
three vectors ~i, ~j, and ~k, each of length 1, to point along the direction of the
positive numbers along each of our three axes as shown in Figure 10.2.7.

3
Figure 10.2.7
Now, let ~v be a vector in the three dimensional system with tail A = (a, b, c)
and tip B = (a0 , b0 , c0 ).

Figure 10.2.8
According to Figure 10.2.8 we can write

~v = AB = AC + CB.

But AC = (a0 a)~i + (b0 b)~j and CB = (c0 c)~k. Hence

AB = (a0 a)~i + (b0 b)~j + (c0 c)~k.

We call < a0 a, b0 b, c0 c > the components of ~v .


Now, if the tail of a vector is the origin of the coordinate system then we call
the vector a position vector. In this case, if A = (a, b, c) are the coordinates
of the tip point of the vector then we can write

~v = OA = a~i + b~j + c~k

or alternatively, ~v =< a, b, c > .

4
Example 10.2.1
Find the components of the vectors ~i, ~j, ~k.

Solution.
We have ~i =< 1, 0, 0 >, ~j =< 0, 1, 0 >, and ~k =< 0, 0, 1 >

Expressing vectors in terms of their components allows us to manipulate them


algebraically. In particular, we can define the operations of addition, subtrac-
tion, and scalar multiplication in terms of the vector components. If ~u =<
a, b, c >= a~i + b~j + c~k, ~v =< a0 , b0 , c0 >= a0~i + b0~j + c0~k and is a scalar then

~u + ~v =(a + a0 )~i + (b + b0 )~j + (c + c0 )~k =< a + a0 , b + b0 , c + c0 > .


~u ~v =(a a0 )~i + (b b0 )~j + (c c0 )~k =< a a0 , b b0 , c c0 > .
~u =a~i + b~j + c~k =< a, b, c > .

Also, the magnitude of a vector can be expressed in terms of its components.



Consider a vector ~v =< a, b, c >= a~i + b~j + c~k. Let OP be a position vector with
the same direction as ~v and the same magnitude. This implies that P = (a, b, c).
But then the magnitude of ~v is just the distance between the origin and the point
P . That is, p
||~v || = ||OP || = a2 + b2 + c2 .

Remark 10.2.1
All the above apply as well for the 2-D case by just letting z = 0.

Example 10.2.2
Perform the operation (4~i + 2~j) (3~i ~j).

Solution.
We have

(4~i + 2~j) (3~i ~j) = (4 3)~i + (2 (1))~j = ~i + 3~j

Example 10.2.3
Find the length of the vector ~v = ~i ~j + 2~k.

Solution. p
The length is ||~v || = 12 + (1)2 + 22 = 6

Example 10.2.4
Show that the vectors ~u = ~i ~j + 3~k and ~v = 4~i 4~j + 12~k are parallel.

Solution.
Since ~v = 4~u, we conclude that the two vectors are parallel

5
Unit Vectors
By a unit vector we mean any vector of magnitude equals to 1. From any
non-zero vector ~v , we can obtain a unit vector by setting

~v
~u = .
||~v ||

To see this, write ~v = a~i + b~j + c~k. Then ||~v || = a2 + b2 + c2 . Moreover,

a ~i + b ~j + c ~k.
~u =
a2 2
+b +c 2 2 2
a +b +c 2 a + b2 + c 2
2

Therefore,
r r
a2 b2 c2 a2 + b2 + c2
||~u|| = 2 2 2
+ 2 2 2
+ 2 = = 1.
a +b +c a +b +c a + b2 + c2 a2 + b2 + c2
Example 10.2.5
Find a unit vector from the point P (1, 2) and toward the point Q(4, 6).

Solution.

Let ~v = P Q = (4 1)~i + (6 2)~j = 3~i + 4~j. Then
p
||~v || = 32 + 42 = 5.

Hence, a unit vector is


~v 3 4~
~u = = ~i + j
||~v || 5 5

Properties of Vector Arithmetic


The arithmetic operations on vectors introduced earlier in this section satisfy the
following properties. These properties are valid for any number of components.

Theorem 10.2.1
If ~u, ~v , and w
~ are three vectors (with the same number of components) and a
and b are two scalars then we have the following properties:

~u + ~v =~v + ~u
~u + (~v + w)
~ =(~u + ~v ) + w
~
~u + ~0 =~u
1~u =~u
(a + b)~u =a~u + b~v
a(~u + ~v ) =a~u + a~v .

Proof.
These properties can be easily proved using components. We will prove the first

6
one leaving the rest for the reader. Write ~u = u1~i + u2~j and ~v = v1~i + v2~j. Then

~u + ~v =u1~i + u2~j + v1~i + v2~j


=u1~i + v1~i + u2~j + v2~j
=v1~i + u1~i + v2~j + u2~j
=v1~i + v2~j + u1~i + u2~j
=~v + ~u

Example 10.2.6
Find the angle between the vector 8~i + 6~j and the positive x axis.

Solution.
Let be the angle between the given vector and the positive xaxis as shown
in Figure 10.2.9

Figure 10.2.9
6 3
= = tan1 34 36.9

We have: tan = 8 = 4

7
Arkansas Tech University
MATH 2934: Calculus III
Dr. Marcel B Finan

10.3 Multiplication of Vectors: The Scalar or Dot Product


Up to this point we have defined what vectors are and discussed basic notation
and properties. We have also defined basic operations on vectors such as addi-
tion, subtraction, and scalar multiplication.
Now, is there such thing as multiplying a vector by another vector? The answer
is yes. As a matter of fact there are two types of vector multiplication. The first
one is known as scalar or dot product1 and produces a scalar; the second is
known as the vector or cross product and produces a vector. In this section
we will discuss the former one leaving the latter one for the next section.
One of the motivation for using the dot product is the physical situation to
which it applies, namely that of computing the work done on an object by a
given force over a given distance, as shown in Figure 10.3.1.

Figure 10.3.1
Indeed, the work W is given by the expression

W = ||F~ || ||P Q|| cos

where ||F~ || cos is the component of F~ in the direction of P Q.
Thus, we define the dot product of two vectors ~u and ~v to be the number

~u ~v = ||~u|| ||~v || cos , 0

where is the angle between the two vectors as shown in Figure 10.3.2.

Figure 10.3.2
The above definition is the geometric definition of the dot product. We next
provide an algebraic way for computing the dot product. Indeed, let ~u = u1~i +
u2~j +u3~k and ~v = v1~i+v2~j +v3~k. Then ~v ~u = (v1 u1 )~i+(v2 u2 )~j +(v3 u3 )~k.
1 Also called inner product.

1
Moreover, we have

||~u||2 =u21 + u22 + u23


||~v ||2 =v12 + v22 + v32
||~v ~u||2 =(v1 u1 )2 + (v2 u2 )2 + (v3 u3 )2
=v12 2v1 u1 + u21 + v22 2v2 u2 + u22 + v32 2v3 u3 + u23 .

Now, applying the Law of Cosines to Figure 10.3.3 we can write

||~v ~u||2 = ||~u||2 + ||~v ||2 2||~u|| ||~v || cos .

Thus, by substitution we obtain

v12 2v1 u1 +u21 +v22 2v2 u2 +u22 +v32 2v3 u3 +u23 = u21 +u22 +u23 +v12 +v22 +v32 2||~u|| ||~v || cos

or
||~u|| ||~v || cos = u1 v1 + u2 v2 + u3 v3
so that we can define the dot product algebraically by

~u ~v = u1 v1 + u2 v2 + u3 v3 .

Figure 10.3.3

Example 10.3.1
Compute the dot product of ~u = 1 ~i + 1 ~
2 2
j + 12 ~k and ~v = 21~i + 12~j + ~k and the
angle between these vectors.

Solution.
We have
1 1 1 1 1 1 1 1
~u ~v = + + 1 = + + = 2.
2 2 2 2 2 2 2 2 2 2
We also have
 2  2  2
1 1 1 3
||~u||2 = + + =
2 2 2 2
 2  2
1 1 3
||~v ||2 = + +1= .
2 2 2

2
Thus,
~u ~v 2 2
cos = = .
||~u|| ||~v || 3
Hence,
!
2 2
= cos1 0.34 rad 19.5
3

Remark 10.3.1
The algebraic definition of the dot product extends to vectors with any number
of components.

Next, we discuss few properties of the dot product.

Theorem 10.3.1
For any vectors ~u, ~v , and w ~ and any scalar we have
(i) Commutative law: ~u ~v = ~v ~u.
(ii) Distributive law: (~u + ~v ) w ~ = ~u w
~ + ~v w.
~
(iii) ~u (~v ) = (~u) ~v = (~u ~v ).
(iv) Magnitude: ||~u||2 = ~u ~u.
(v) Two nonzero vectors ~u and ~v are orthogonal or perpendicular if and only
if ~u ~v = 0.
(vi)) Two nonzero vectors ~u and ~v are parallel if and only if ~u ~v = ||~u|| ||~v ||.
(vii) ~0 ~v = 0.

Proof.
Write ~u = u1~i + u2~j + u3~k, ~v = v1~i + v2~j + v3~k, and w ~ = w1~i + w2~j + w3~k. Then
(i) ~u ~v = u1 v1 + u2 v2 + u3 v3 = v1 u1 + v2 u2 + v3 u3 = ~v ~u since product of
numbers is commutative.
(ii) (~u + ~v ) w~ = ((u1 + v1 )~i + (u2 + v2 )~j + (u3 + v3 )~k) (w1~i + w2~j + w3~k) =
(u1 + v1 )w1 + (u2 + v2 )w2 + (u3 + v3 )w3 = u1 w1 + u2 w2 + u3 w3 + v1 w1 + v2 w2 +
v3 w3 = ~u w ~ + ~v w.
~
(iii) ~u (~v ) = (u1~i + u2~j + u3~k) (v1~i + v2~j + v3~k) = u1 v1 + u2 v2 + u3 v3 =
(u1 v1 + u2 v2 + u3 v3 ) = (~u ~v ).
(iv) ||~u||2 = ~u ~u cos 0 = ~u ~u.
(v) If ~u and ~v are perpendicular then the cosine of their angle is zero and so
the dot product is zero. Conversely, if the dot product of the two vectors is
zero then the cosine of their angle is zero and this happens only when the two
vectors are perpendicular.
(vi) If ~u and ~v are parallel then the cosine of their angle is either 1 (vectors
are of the same direction) or 1 (vectors are of opposite directions). That is,
~u ~v = ||~u|| ||~v ||. Conversely, if ~u ~v = ||~u|| ||~v || then cos = 1 and this
implies that either = 0 or = . In either case, the two vectors are parallel.
(vii) In 3-D, ~0 =< 0, 0, 0 > and ~v =< a, b, c > so that ~0 ~v = (0 a)~i + (0
b)~j + (0 c)~k = ~0

3
Remark 10.3.2
Note that the unit vectors ~i, ~j, ~k associated with the coordinate axes satisfy the
equalities
~i ~i = ~j ~j = ~k ~k = 1 and ~i ~j = ~j ~k = ~i ~k = 0.

Example 10.3.2
(a) Show that the vectors ~u = 3~i 2~j and ~v = 2~i + 3~j are perpendicular.
(b) Show that the vectors ~u = 2~i + 6~j 4~k and ~v = 3~i 9~j + 6~k are parallel.

Solution.
(a) We have: ~u ~v = 3(2) 2(3) = 0. Hence ~u is perpendicular to ~v .
(b) We have:

~u ~v 2(3) + (6)(9) 4(6)


cos = = p p = 1.
||~u||||~v || [ 22 + (6)2 + (4)2 ][ (3)2 + (9)2 + 62 ]

Hence, = so that the two vectors are parallel. Another way to see that the
vectors are parallel is to notice that ~u = 32 ~v

Projection of a vector onto a line


The orthogonal projection of a vector along a line is obtained by taking a
vector with same length and direction as the given vector but with its tail on
the line and then dropping a perpendicular onto the line from the tip of the
vector. The resulting vector on the line is the vectors orthogonal projection or
simply its projection. See Figure 10.3.4.

Figure 10.3.4

Now, if ~u is a unit vector along the line of projection and if ~vparallel is the vector
projection of ~v onto ~u then

~vparallel = (||~v || cos )~u = (~v ~u)~u.

See Figure 10.3.5. Also, the component perpendicular to ~u is given by

~vperpendicular = ~v ~vparallel .

4
Figure 10.3.5
We call Comp~u~v = ~v ~u the scalar projection of ~v onto ~u. We call the vector
Proj~u~v = ~vparallel the vector projection of ~v onto ~u.
It follows that the vector ~v can be written in terms of ~vparallel and ~vperpendicular
~v = ~vparallel + ~vperpendicular .
Example 10.3.3
Write the vector ~v = 3~i + 2~j 6~k as the sum of two vectors, one parallel, and
one perpendicular to w~ = 2~i 4~j + ~k.
Solution.
w
~
Let ~u = ||w|| 2 ~i 4 ~ 1 ~
~ = 21 21
j + 21
k. Then,
 
6 8 6 16~ 32~ 8
~vparallel = (~v ~u)~u = ~u = i + j ~k.
21 21 21 21 21 21
Also,
     
16 ~ 32 ~ 8 ~
~vperpendicular =~v ~vparallel = 3 + i+ 2 j + 6 + k
21 21 21
79 10 118 ~
= ~i + ~j k.
21 21 21
Hence,
~v = ~vparallel + ~vperpendicular
Example 10.3.4
Find the scalar projection and vector projection of ~u =< 1, 1, 2 > onto ~v =<
2, 3, 1 > .
Solution.
We have
~u ~v 1(2) + (1)(3) + 2(1) 3
comp~v ~u = = p =
||~v || 2 2
(2) + 3 + 1 2 14
~u ~v ~v
Proj~v ~u =
||~v || ||~v ||
3 3 9 3
= ~v = ~i + ~j + ~k
14 7 14 14

5
Applications
As pointed out earlier in the section, scalar products are used in Physics. For
instance, in finding the work done by a force applied on an object.

Example 10.3.5
A wagon is pulled a distance of 100 m along a horizontal path by a constant
force of 70 N. The handle of the wagon is held at an angle of 35 above the
horizontal. Find the work done by the force.

Solution.
The work done is

W = F d cos 35 = 70(100) cos 35 5734 J

6
Arkansas Tech University
MATH 2934: Calculus III
Dr. Marcel B Finan

10.4 Multiplying Vectors: The Cross Product


In this section we discuss another way of multiplying two vectors to obtain a
vector, the cross product. We should note that the cross product requires
both of the vectors to be three dimensional vectors.
Now, any two three dimensional vectors determine a parallelogram. We will see
below that the definition of cross product involves this parallelogram.
Consider first the parallelogram obtained by two vectors ~u and ~v and let be
the angle between these two vectors as shown in Figure 10.4.1.

Figure 10.4.1

The area of this parallelogram is given by

Area of parallelogram = Base Height = ||~v || ||~u|| sin .

We define the cross product of two vectors ~u and ~v to be the vector which
is perpendicular to both ~u and ~v with a magnitude equal to the area of the
parallelogram they span. The corresponding formula is

~u ~v = (||~v || ||~u|| sin )~n

where 0 is the angle between the two vectors and ~n is a unit vector
perpendicular to both ~u and ~v .
The first problem that we encounter here is the normal vector ~n. For any two
vectors ~u and ~v , if ~n is a normal vector so is ~n. So which vector is the correct
one? This is determined by the following rule, known as the right-hand rule:
The orientation of ~n is determined by placing ~u and ~v tail-to-tail, flattening the
right hand, extending it in the direction of ~u, and then curling the fingers in the
direction leading to ~v . The thumb then points in the direction of ~n. See Figure
10.4.2.

1
Figure 10.4.2
It follows from the definition, that if ~u and ~v are parallel, then sin = 0 and
therefore ~u ~v = ~0. In particular, ~u ~u = ~0.

Example 10.4.1
Find all possible cross products of the unit vectors ~i, ~j, and ~k.

Solution.
We have
~i ~j =||~i|| ||~j|| sin ( )~k = ~k
2
~j ~k =||~j|| ||~k|| sin ( )~i = ~i
2
~k ~i =||~k|| ||~i|| sin ( )~j = ~j
2
~j ~i =||~j|| ||~i|| sin ( )(~k) = ~k
2
~k ~j =||~k|| ||~j|| sin ( )(~i) = ~i
2
~i ~k =||~i|| ||~k|| sin ( )(~j) = ~j.
2
There is an easy way to deal with the signs. Consider the diagram in Figure
10.4.3.

Figure 10.4.3

2
Any product that moves around the diagram clockwise takes a plus sign, and
any product that moves around the diagram counterclockwise takes a minus
sign.
Finally, ~i ~i = ~j ~j = ~k ~k = ~0 since the angle between a vector with itself is
zero

So far the cross product has been defined geometrically. How can we find ~u ~v
if both vectors are given in components? That is, if ~u = u1~i + u2~j + u3~k and
~v = v1~i + v2~j + v3~k then what is ~u ~v ?
Lets first introduce the determinant of three rows and three columns:

x y z

u1 u2 u3 = (u2 v3 v2 u3 )x (u1 v3 v1 u3 )y + (u1 v2 u2 v1 )z.

v1 v2 v3
By substitution one can easily find that

u1 u2 u3

u1 u2 u3 = (u2 v3 v2 u3 )u1 (u1 v3 v1 u3 )u2 + (u1 v2 u2 v1 )u3 = 0

v1 v2 v3
and

v1 v2 v3

u1 u2 u3 = (u2 v3 v2 u3 )v1 (u1 v3 v1 u3 )v2 + (u1 v2 u2 v1 )v3 = 0.

v1 v2 v3

~ = (u2 v3 v2 u3 )~i (u1 v3 v1 u3 )~j + (u1 v2 u2 v1 )~k we see


Thus, by letting w
that
~ ~u = 0 and w
w ~ ~v = 0.
Hence, w ~ and ~u ~v are
~ is perpendicular to both ~u and ~v . It follows that w
~ = (~u ~v ).
parallel. Hence, w
On the other hand, we have the following
~ 2 =(u2 v3 v2 u3 )2 + (u1 v3 v1 u3 )2 + (u1 v2 u2 v1 )2
||w||
=(u22 v32 2u2 v3 v2 u3 + v22 u23 ) + (u21 v32 2u1 v3 v1 u3 + v12 u23 )
+(u21 v22 2u2 v1 v2 u1 + v12 u22 )
=u22 v32 + v22 u23 + u21 v32 + v12 u23 + u21 v22 + v12 u22
2u2 v3 v2 u3 2u1 v3 v1 u3 2u2 v1 v2 u1
=(u21 + u22 + u23 )(v12 + v22 + v32 ) (u1 v1 + u2 v2 + u3 v3 )2
=||~u||2 ||~v ||2 (~u ~v )2
and
||~u ~v ||2 =||~u||2 ||~v ||2 sin2
=||~u||2 ||~v ||2 (1 cos2 )
=||~u||2 ||~v ||2 (~u ~v )2 .

3
We conclude that = 1. But by checking the formula of w ~ when ~u = ~i and
~v = ~j we notice that w~ = ~k and ~u ~v have the same direction. Hence, = 1
and so ~u ~v = w.
~
From the above discussion, a convenient way to remember the algebraic cross
product is by means of the determinant of the following matrix:

~k

~i ~j
u2 u3 u1 u3 u1 u2
~ ~ ~k

~u ~v = u1 u2 u3 =
i
j+
v1 v2 v3 v2 v3 v1 v3 v1 v2

Example 10.4.2
Find the cross product of ~u = 3~i 3~j + ~k and ~v = 4~i + 9~j + 2~k and check that
the cross product is perpendicular to both ~u and ~v .

Solution.
We have
~j ~k

~i

~u ~v = 33 1
4 9 2

3 1 ~ 3
1 ~ 3 3 ~
= i j + k
9 2 4 2 4 9
= 15~i 2~j + 39~k

Moreover,
~u (~u ~v ) = (3)(15) 3(2) + 39 = 0
and
~v (~u ~v ) = (4)(15) + 9(2) + 2(39) = 0.
Thus, ~u ~v is perpendicular to both ~u and ~v

Remark 10.4.1
The identity ||~u ~v || = ||~u||2 ||~v ||2 (~u ~v )2 is knwon as Lagranges identity.
Since ||~u||2 ||~v ||2 (~u ~v )2 = ||~u ~v || 0, we obtain the inequality

~u ~v ||~u|| ||~v ||.

This is known as Schwarzs inequality

The following theorem lists some of the properties of cross product.

Theorem 10.4.1
For vectors ~u, ~v , w
~ and scalar we have
(i) ~u ~v = ~v ~u
(ii) (~u) ~v = (~u ~v ) = ~u (~v )
(iii) ~u (~v + w)
~ = ~u ~v + ~u w ~
(iv) ~u (~v w)
~ = (~u ~v ) w
~

4
Proof.
Let ~u = u1~i + u2~j + u3~k, ~v = v1~i + v2~j + v3~k, and w
~ = w1~i + w2~j + w3~k.
(i) We have

~u ~v = (u2 v3 v2 u3 )~i (u1 v3 u3 v1 )~j + (u1 v2 u2 v1 )~k

and
~v ~u = (v2 u3 u2 v3 )~i (v1 u3 v3 u1 )~j + (v1 u2 v2 u1 )~k.
Hence, ~u ~v = ~v ~u
(ii) We have

(~u) ~v =[(u2 )v3 v2 (u3 )]~i [(u1 )v3 (u3 )v1 ]~j + [(u1 )v2 (u2 )v1 ]~k
=[(u2 v3 v2 u3 )~i (u1 v3 u3 v1 )~j + (u1 v2 u2 v1 )~k]
=(~u ~v ).

(iii) We leave this to the reader.


(iv) We have

~ =(u1~i + u2~j + u3~k) ((v2 w3 w2 v3 )~i (v1 w3 v3 w1 )~j + (v1 w2 v2 w1 )~k)


~u (~v w)
=u1 (v2 w3 w2 v3 ) u2 (v1 w3 v3 w1 ) + u3 (v1 w2 v2 w1 )
=u1 v2 w3 u1 w2 v3 u2 v1 w3 + u2 v3 w1 + u3 v1 w2 u3 v2 w1
=(u2 v3 v2 u3 )w1 (u1 v3 u3 v1 )w2 + (u1 v2 u2 v1 )w3
=((u2 v3 v2 u3 )~i (u1 v3 u3 v1 )~j + (u1 v2 u2 v1 )~k) w
~
=(~u ~v ) w
~

Example 10.4.3
Find the area of the parallelogram with edges ~u = 3~i3~j +~k and ~v = 4~i+9~j +2~k

Solution.
From Example 10.4.2, we found that

~u ~v = 15~i 2~j + 39~k.

Thus
p
Area of Parallelogram = ||~u ~v || = (15)2 + (2)2 + 392 = 5 70

Volume of a Parallelepiped
Consider the parallelepiped spanned by the vectors ~a, ~b, and ~c as shown in Figure
10.4.4.

5
Figure 10.4.4
The volume of this figure is the area of the base times the height. We already
know the area of the parallelogram base: ||~a ~b||. The height is the component
of ~c in the direction normal to the base, i.e., in the direction of ~a ~b. Hence the
height is ||~c|| | cos | (the absolute value is needed for if > 2 then cos < 0).
The volume of the parallelepiped is therefore

Volume = ||~a ~b|| ||~c|| cos = |(~a ~b) ~c|.

But recall that the components of ~a ~b are



a a3
~
~a b = 2 ~i a1 a3 ~j + a1 a2 ~

k.
b2 b3 b1 b3 b1 b2
Hence,

a2 a3 a1 a3 a1 a2
(~a ~b) ~c =

c 1 c2 + c
b2 3

b2 b3 b1 b3 b1


c1 c2 c3
= a1 a2 a3
b1 b2 b3
and the volume of the parallelepiped is just the absolute value of this determi-
nant.

Remark 10.4.2
A simple algebra shows that


c1 c2 c3 a1
a2 a3


a1 a2 a3 = b1
b2 b3 .

b1 b2 b3 c1 c2 c3

Example 10.4.4
Find the volume of the parallelepiped spanned by the vectors ~a =< 2, 3, 1 >
, ~b =< 0, 4, 0 >, and ~c =< 1, 3, 3 > .

6
Solution.
Since

2 3 1
~

(~a b) ~c = 0 4 0
1 3 3
= 2(12 0) 3(0 0) + 1(0 + 4) = 20,
the volume of the parallelepiped is | 20| = 20

Application: Torque
A torque is the tendency of a force to rotate an object about an axis. Just as
a force is a push or a pull, a torque can be thought of as a twist to an object.
For example, pushing or pulling the handle of a wrench connected to a nut or
bolt produces a torque (turning force) that loosens or tightens the nut or bolt.
See Figure 10.4.5.

Figure 10.4.5
Mathematically, torque is defined as the cross product of the position vector of
the point where the force is applied and the force vector, which tends to produce
rotation.
Let ~ denote the torque vector, ~r the vector with tail the origin of the coordinates
system and its tip is the point where the acting force is applied, F~ the acting
force vector applied at the tip of ~r. We have
~ = ~r F~ = (||~r||||F~ || sin )~n
where is the angle between the position and force vectors and ~n is a vector
normal to the two vectors. Observe that the only component of F~ that can
cause a rotation is the one perpendicular to ~r, that is, ||F~ || sin .
Example 10.4.5
A wrench 30 cm long lies along the positive yaxis and grips a bolt at the
origin. A force is applied in the direction < 0, 3, 4 > at the end of the wrench.
Find the magnitude of the force needed to supply 100 N m of torque to the
bolt.
Solution.
||~
||
We have ~r =< 0, 0.3, 0 > and ||~ || = 100. Thus, ||F~ || = r ||| sin | .
||~ Now,
< 0, 0.3, 0 > < 0, 3, 4 >
cos = = 0.6.
|| < 0, 0.3, 0 > |||| < 0, 3, 4 > ||

7
Hence, = cos1 (0.6) 53.1 . Thus, ||F~ || = 100
0.3 sin 53.1 417 N

8
Arkansas Tech University
MATH 2934: Calculus III
Dr. Marcel B Finan

10.5 Equations of Lines and Planes


We first start this section by deriving the vector and parametric equations
of a straight line in three dimensional space. In 3-D, like in 2-D, a line is
uniquely determined when one point on the line and a direction vector are
given. To this end, let P0 (x0 , y0 , z0 ) be a fixed point on a straight line (L)
and let ~v =< a, b, c > be a vector parallel to (L), i.e., a direction vector1

of (L). Let P (x, y, z) be an arbitrary point of (L). Let r~0 = OP0 and ~r = OP
be position vectors as shown in Figure 10.5.1.

Figure 10.5.1

As we can see from Figure 10.5.1, a necessary and sufficient condition for a

point P to be on the line (L) is that P0 P =< x x0 , y y0 , z z0 > is

parallel to ~v . That is, P0 P = t~v for some scalar t. From Figure 10.5.1, we

can write r~0 + P0 P = ~r. Thus,

~r = r~0 + t~v . (10.5.1)


1
The numbers a, b, and c are called direction numbers

1
Equation (10.5.1) is called the vector equation of the line (L). The pa-
rameter t can be any real number. As it varies, the point P moves along
the line. When t = 0, P is the same as P0 . When t > 0, P is away from
P0 in the direction of ~v and when t < 0, P is away from P0 in the direction
opposite to ~v .
Now, in terms of components, Equation (10.5.1) leads to the vector equality
< x, y, z >=< x0 + ta, y0 + tb, z0 + tc > or equivalently

x = x0 + at, y = y0 + bt, z = z0 + ct. (10.5.2)

Equations (10.5.2) are called the parametric equations of the line (L).
Also, from Equations (10.5.2), we can write
x x0 y y0 z z0
= = . (10.5.3)
a b c
Equations (10.5.3) are called the symmetric equations of the line (L).

Example 10.5.1
Find vector, parametric and symmetric equations of the line passing through
the point P (1, 4, 2) and parallel to ~v =< 1, 2, 3 > .

Solution.
The vector equation of the line is

~r = ~i + 4~j + 2~k + t(~i + 2~j + 3~k) = (1 + t)~i + (4 + 2t)~j + (2 + 3t)~k.

The parametric equations are

x = 1 + t, y = 4 + 2t, z = 2 + 3t.

The symmetric equations are


y4 z2
x+1= =
2 3
Remark 10.5.1
Vector, parametric, and symmetric equations of a line are not unique since
they depend on a specific given point on the a line and a specific vector
parallel to the line.

2
Example 10.5.2
Find the vector equation of the line segment with tail P0 and tip P1 .

Solution.

Let P a point on the given line segment. Then P0 P and P0 P1 are parallel so

that P0 P = tP0 P1 for 0 t 1. On the other hand, using position vectors,

we can write r~0 + P0 P = ~r or

r~0 + tP0 P1 = ~r. (10.5.4)

But r~0 + P0 P1 = r~1 . Hence, P0 P1 = r~1 r~0 . Substituting this into Equation
(10.5.4), we find the vector equation of the requested line segment

~r = (1 t)~ r1 , 0 t 1
r0 + t~

Example 10.5.3
Find the vector and parametric equations of the line segment with tail
P0 (2, 4, 3) and tip P1 (3, 1, 1).

Solution.
The vector equation is

~r = (1t)(2~i+4~j 3~k)+t(3~i~j +~k) = (2+t)~i+(45t)~j +(4t3)~k, 0 t 1.

The parametric equations are

x = 2 + t, y = 4 5t, z = 4t 3, 0 t 1

Example 10.5.4
Two lines are said to be skew if they do not cross and they are not parallel
(and hence they do not belong to the same plane). Show that the lines (L1 )
and (L2 ) with parametric equations

x = 1 + t, y = 2 + 3t, z = 4 t

and
x = 2s, y = 3 + s, z = 3 + 4s
respectively, are skew lines.

3
Solution.
The vector direction to (L1 ) is v~1 =< 1, 3, 1 > . The vector direction to
(L2 ) is v~2 =< 2, 1, 4 > . Since

v~1 v~2 1
cos (v~1 , v~2 ) = = 6= 0 and 1
||v~1 ||||v~2 || 231
so that the two lines are not parallel. If the two lines intersect, we should
have
1 + t = 2s, 2 + 3t = 3 + s, 4 t = 3 + 4s.
Solving the first two equations, we find t = 11
5
and s = 85 . But these values
do not work for our third equation. This, shows that the two lines do not
intersect. Hence, we conclude that the two lines are skew

Equations of Planes
Now, we will try to find the equation of a plane given a point in the plane
and a vector normal to the plane. But first we need to define by what we
mean by a normal vector.
A normal vector to a plane is a vector that is orthogonal to every vector of
the plane. If ~n = a~i + b~j + c~k is normal to a given plane and P0 = (x0 , y0 , z0 )
is a fixed point in the plane then for any point P = (x, y, z) in the plane the

vectors ~n and P0 P are orthogonal (i.e., perpendicular) (See Figure 10.5.2.)

Figure 10.5.2

In terms of dot product this means that



P0 P ~n = 0.

Resolving P0 P in terms of components we find

P0 P = (x x0 )~i + (y y0 )~j + (z z0 )~k.

4

In this case, the dot product P0 P ~n = 0 leads to the equation
a(x x0 ) + b(y y0 ) + c(z z0 ) = 0 (10.5.5)
which is the equation of our required plane.
Example 10.5.5
Write an equation of a plane through P (1, 0, 1) with normal vector ~n =
2~i + 2~j ~k.
Solution.
The equation is
2(x 1) + 2(y 0) + (1)(z + 1) = 0 or z = 2x + 2y 3
By collecting terms in equation (10.5.5), we can rewrite the equation of the
plane in the form
ax + by + cz + d = 0 (10.5.6)
where d = (ax0 + by0 + cz0 ). Equation (10.5.6) is called a linear equation
in x, y, and z.
Example 10.5.6
Find a normal vector to the plane x + 3y + 2z = 7.
Solution.
Since the coefficients of ~i, ~j, and ~k in a normal vector are the coefficients of
x, y, and z in the equation of the plane, a normal vector is ~n = ~i + 3~j + 2~k
Example 10.5.7 (The Equation of a Plane Through Three Points)
Find the equation for the plane through the points P0 = (0, 1, 7), P1 =
(3, 1, 9), and P2 = (0, 5, 8).
Solution.

We have P0 P1 = (3 0)~i + (1 1)~j + (9 + 7)~k = 3~i 2~k. Similarly,

P0 P2 = 6~j ~k. Thus, the vector ~n = P0 P1 P0 P2 = 12~i + 3~j 18~k is
normal to the plane at P0 . Hence, the equation of the plane is

~n P0 P = 0
where P = (x, y, z). Hence,
12x + 3(y 1) 18(z + 7) = 0
or
12x + 3y 18z 129 = 0

5
Example 10.5.8 (Parallel planes)
Two planes are parallel if their normal vectors are parallel. Show that the
planes x + 2y 3z = 4 and 2x + 4y 6z = 3 are parallel.
Solution.
The vector normal to the plane x + 2y 3z = 4 is ~v =< 1, 2, 3 > . The
vector normal to the plane 2x + 4y 6z = 3 is w
~ =< 2, 4, 6 >= 2~v . Since
~v and w~ are parallel so are the two planes
Example 10.5.9 (Intersecting planes)
When two planes intersect their intersection is a straight line. The angle
between the two planes is the angle between their normal vectors as shown
in Figure 10.5.3.

Figure 10.5.3
(a) Find the angle of intersection of the two planes x + y + z = 1 and
x 2y + 3z = 1.
(b) Find the symmetric equations of the line of intersection.
Solution.
(a) The angle between the two planes is
   
1 < 1, 1, 1 > < 1, 2, 3 > 1 2
= cos = cos 72 .
|| < 1, 1, 1 > |||| < 1, 2, 3 > || 42
(b) To find a point on the line of intersection, we find a point where the line
intersect the xyplane. In this case, we set z = 0 and obtain the system of
equations x + y = 1 and x 2y = 1. Solving this system by elimination, we
find x = 1 and y = 0. Hence, the point (1, 0, 0) is on the line of intersection.
Next, a direction vector is the vector
~i ~j ~k

< 1, 1, 1 > < 1, 2, 3 >= 1 1 1 = 5~i 2~j 3~k.



1 2 3

6
Hence, the symmetric equations of the line of intersection are
x1 y z
= =
5 2 3
We conclude this section, by finding the distance of a point P1 (x1 , y1 , z1 ) to a
plane with linear equation ax + by + cz + d = 0. To this end, let P0 (x0 , y0 , z0 )
be any point in the plane. According to Figure 10.5.4, the distance from P1

to the plane is the absolute value of the scalar projection of P0 P1 onto the
normal vector ~n =< a, b, c > .

Figure 10.5.4

Hence,

P P ~n |a(x x ) + b(y y ) + c(z z )|
0 1 1 0
D = = 1 0 1 0
||~n|| 2 2
a +b +c 2

|(ax1 + by1 + cz1 ) (ax0 + by0 + cz0 )|


=
a2 + b 2 + c 2
|ax1 + by1 + cz1 + d|
=
a2 + b 2 + c 2
where we used the fact that the point P0 is in the plane so that ax0 + by0 +
cz0 = d.

Example 10.5.10
Find the distance between the parallel planes z = x+2y+1 and 3x+6y3z =
4.

Solution.
The vector normal to the plane z = x + 2y + 1 is ~v =< 1, 2, 1 > . The
vector normal to the plane 3x + 6y 3z = 4 is w
~ =< 3, 6, 3 >= 3~v . Since
~v and w~ are parallel so are the two planes.

7
A point must be found that lies on one of the planes. When x = y = 0 in
the plane z = x + 2y + 1, there exists the point (0, 0, 1). To find the distance
between the planes we can now use the equation for the distance between a
point and a plane.

|ax1 + by1 + cz1 + d| |3(0) + 6(0) 3(1) 4| 7


D= = p =
2 2
a +b +c 2 2 2
3 + 6 + (3) 2 3 6

8
Arkansas Tech University
MATH 2934: Calculus III
Dr. Marcel B Finan

10.6 Cylinders and Quadric Surfaces


We start this section with a review of quadratic curves in 2-D known as conic
sections. In IR2 , a quadratic equation of the form y = ax2 gives one of the
two curves shown in Figure 10.6.1. Each curve is called a parabola. The
graph of any quadratic equation is obtained by a sequence of transformations
of the above parabola. Indeed, by the method of completing the square, we
have 2
4ac b2

2 b
f (x) = ax + bx + c = a x + + .
2a 4a

Figure 10.6.1
2
x2
The graph of a quadratic equation of the form a2
+ yb2 = 1 is called an ellipse
as shown in Figure 10.6.2.

Figure 10.6.2
x2 y2 y2 x2
The graph of a quadratic equation of the form a2
b2
= 1 or b2
a2
= 1 is
called a hyperbola as shown in Figure 10.6.3.

1
Figure 10.6.3

By a surface we mean the graph of an equation in the variables x, y, z :


f (x, y, z) = 0. The intersection of a plane parallel to a coordinate plane with
a surface is called a trace or a cross-section. For example, the trace of the
sphere x2 + y 2 + z 2 = 100 with a plane parallel to the xyplane is a circle.
By a cylindrical surface or a cylinder we mean a surface that consists of
all lines (called rulings) that are parallel to a given line and pass through a
plane curve (called the generator).

Example 10.6.1
2
x2
Sketch the graph of each of the following cylindrical surfaces: (a) a2
+ yb2 = 1
2 2
(b) x = ay 2 , a > 0 (c) xa2 yb2 = 1.

Solution.
(a) This surface is called an elliptic cylinder. The rulings are parallel to
the missing variable which is in this case the zaxis. The generator is an
ellipse. See Figure 10.6.4(a). Vertical traces are union of two parallel lines
whereas horizontal traces are ellipses.
(b) This surface is called a parabolic cylinder. The rulings are parallel
to the missing variable which is in this case the zaxis. The generator is
a parabola. See Figure 10.6.4(b). Horizontal traces are parabolas. Vertical
traces are the union of two parallel lines. Notice that the axis of symmetry
is the variable with second power.
(c) This surface is called a hyperbolic cylinder. The rulings are parallel
to the missing variable which is in this case the zaxis. The generator is
a hyperbola. See Figure 10.6.4(c). Horizontal traces are hyperbolas. The
vertex of the generator is on the axis with a plus sign which is in this case
the xaxis

2
Quadric Surfaces
Quadric surfaces are the counterparts in three dimensions of the conic sec-
tions in the plane. A quadric surface is the graph of a second-degree
equation in the variables x, y, z given by

Ax2 + By 2 + Cz 2 + Dxy + Eyz + F xz + Gx + Hy + Iz + J = 0.

There is no way that we can possibly list all of them, but there are some
standard equations so here is a list of some of the more common quadric
surfaces.
2 2 2
An ellipsoid is the surface with equation xa2 + yb2 + zc2 = 1. See Figure 10.6.5.
Intersections with any plane parallel to a coordinate plane (i.e., traces) give
an ellipse.

Figure 10.6.5
Notice that a sphere centered at the origin and with radius a is a special case
of an ellipsoid where a = b = c.

3
2 2
An elliptic paraboloid is the surface with equation zc = xa2 + yb2 . See Figure
10.6.6. Vertical traces are parabolas whereas horizontal traces are ellipses.

Figure 10.6.6
2 2
A hyperbolic paraboloid is the surface with equation zc = xa2 yb2 . See
Figure 10.6.7. vertical traces are parabolas whereas horizontal traces are
hyperbolas. Notice that the shape of the surface near the origin resembles
that of a saddle.

Figure 10.6.7

Notice that the variable raised to the first power indicates the axis of sym-
metry.
2 2 2
A hyperboloid of one sheet is the surface with equation xa2 + yb2 zc2 = 1.
See Figure 10.6.8. Vertical traces are hyperbolas whereas horizontal traces

4
are ellipses. The axis of symmetry corresponds to the variable whose coeffi-
cient is negative.

Figure 10.6.8
2 2 2
A hyperboloid of two sheets is the surface with equation xa2 yb2 zc2 =
1. See Figure 10.6.9. Vertical traces are ellipses whereas horizontal traces
are hyperbolas. The two minus signs indicate two sheets and the axis of
symmetry is the variable with a plus sign.

Figure 10.6.9
2
A cone with axis of symmetry the zaxis is the surface with equation zc2 =
2
x2
a2
+ yb2 . See Figure 10.6.10. Vertical traces are hyperbolas whereas horizontal
traces are ellipses.

5
Figure 10.6.10

Example 10.6.2
Classify the quadric surface x2 + 2z 2 6x y + 10 = 0.

Solution.
By completing the square, we have (x 3)2 + 2z 2 y + 1 = 0 or y 1 =
(x 3)2 + 2z 2 . This is an elliptic paraboloid with axis parallel to the yaxis
and it has been shifted so that its vertex is the point (3, 1, 0). See Figure
10.6.11. The trace in the plane y = k > 1 is (x 3)2 + 2z 2 = k 1. The
trace in the xyplane (i.e., z = 0) is (x 3)2 = y 1

Figure 10.6.11

6
Arkansas Tech University
MATH 2934: Calculus III
Dr. Marcel B Finan

10.7 The Calculus of Vector Functions


A vector function ~r(t) is a rule that assigns to each real number t in its
domain a unique vector ~r(t) = f (t)~i + g(t)~j + h(t)~k in IR3 . The domain
of ~r(t) is the collection of all values of t such that f (t), g(t), and h(t) are
defined. We call f (t), g(t) and h(t) the components of ~r(t).

Example 10.7.1
Find the domain of the vector function ~r(t) = ln t~i + 1 ~
t1
j + e3t~k.

Solution.
The domain is the set of real numbers where all three components are de-
fined. The component f (t) = ln t is defined for all t > 0. The component
1
g(t) = t1 is defined for all t 6= 1 and the component h(t) = e3t is defined for
all real numbers. Hence, the domain of ~r(t) is the set (0, 1) (1, )

Limit of a Vector Function


For a vector function ~r(t) = f (t)~i + g(t)~j + h(t)~k we define the limit
     
lim ~r(t) = lim f (t) i + lim g(t) j + lim h(t) ~k
~ ~
tt0 tt0 tt0 tt0

provided that the limits of the three components exist. Limits of vector
functions obey the same rules as limits of real-valued functions.

Example 10.7.2
Find limt0 ~r(t) where ~r(t) = sin t~
t
i + te2t~j + (2t 3)~k.

Solution.
We have
sin t ~
lim ~r(t) =[lim ]i + [lim te2t ]~j + [lim(2t 3)]~k
t0 t0 t t0 t0

=~i 3~k

Continuity
We say that a vector function ~r(t) is continuous at t0 if and only if

lim ~r(t) = ~r(t0 ).


tt0

Equivalently,
lim ~r(t) = f (t0 )~i + g(t0 )~j + h(t0 )~k.
tt0

1
Example 10.7.3
Show that ~r(t) = cos t~i + et ~j + (3t + 1)~k is continuous at t = 0.
2

Solution.
We have
2
lim ~r(t) = [lim cos t]~i + [lim et ]~j + [lim(3t + 1)]~k = ~i + ~j + ~k = ~r(0)
t0 t0 t0 t0

Space Curves and Continuity


A space curve (C) is the collection of all points (x, y, z) in IR3 where

x = x(t), y = y(t), z = z(t) (10.7.1)

and t varies through an interval I. The equations (10.7.1) are called para-
metric equations and t is called the parameter. We can think of C as be-
ing traced out by a moving particle whose position at time t is (f (t), g(t), h(t)).
There is a close connection between continuous vector functions and space
curves. Let ~r(t) =< f (t), g(t), h(t) > be a position vector with tip the point
(f (t), g(t), h(t) on C. If ~r(t) is continuous then ~r(t) defines a space curve C
that is traced out by the tip of the moving vector ~r(t), as shown in Figure
10.7.1.

Figure 10.7.1

Example 10.7.4
Sketch the curve whose vector function is ~r(t) = cos t~i + sin t~j + t~k.

Solution.
The parametric equations are

x(t) = cos t, y(t) = sin t, z(t) = t.

2
Since x2 + y 2 = 1 the curve lies on the right cylinder x2 + y 2 = 1. Since z = t,
the curve spirals upward as t increases. This curve is called a helix. See
Figure 10.7.2

Figure 10.7.2

Example 10.7.5
Find a vector function that represents the curve of the intersection of the
right cylinder x2 + y 2 = 1 and the plane y + z = 2.

Solution.
Since the plane y + z = 2 is not parallel to the xyplane, the curve of
intersection is an ellipse. This ellipse lies on the cylinder so that x = cos t
and y = sin t. Also, the curve lies in the plane so that z = 2 y = 2 sin t.
Hence, the vector function for the curve of the intersection is

~r(t) = cos t~i + sin t~j + (2 sin t)~k, 0 t 2

Derivatives
Suppose that ~r(t) =< f (t), g(t), h(t) > and f (t), g(t), h(t) are differentiable
functions. Proceeding as in the case of the derivative of one variable, we find
~r(t + h) ~r(t)
r~0 (t) = lim
h0 h
< f (t + h), g(t + h), h(t + h) > < f (t), g(t), h(t) >
= lim
h0 h
f (t + h) f (t) g(t + h) g(t) g(t + h) g(t)
= lim < , , >
h0 h h h
= < f 0 (t), g 0 (t), h0 (t) > .

3
Geometrically, r~0 (t) is the tangent vector to the space curve whose parametric
equations are: x = f (t), y = g(t), z = h(t). See Figure 10.7.3.

Figure 10.7.3

Example 10.7.6
(a) Find the derivative of ~r(t) = cos t~i + sin t~j + t~k.
(b) Find the unit tangent vector at the point where t = 0.

Solution.
(a) We have
r~0 (t) = sin t~i + cos t~j + ~k.
(b) The unit tangent vector at the point where t = 0 is

r~0 (0) ~j + ~k 1 1
T~ (0) = = = ~j + ~k
||r~0 (0)|| 2 2 2

Example 10.7.7
Find parametric equations for the tangent line to the helix ~r(t) = cos t~i +
sin t~j + t~k at the point (0, 1, /2).

Solution.
The parameter value corresponding to the point (1, 0, /2) is t = 2 so the
tangent vector is r~0 2 =< 1, 0, 1 > . The tangent line is the line passing


through (0, 1, /2) and parallel to the vector < 1, 0, 1 > whose parametric
equations are

x = 1, y = t, z = + t
2

4
Remark 10.7.1
Just as for single variable functions, one can take higher order derivatives of
a real valued vector function ~r(t).
The next theorem shows that the differentiation formulas for real-valued
functions have their counterparts for vector-valued functions.
Theorem 10.7.1
Suppose ~u(t) and ~v (t) are differentiable vector functions, c is a scalar, and
f (t) is a real-valued function. Then
(1) [~u ~v ]0 = u~0 v~0
(2) [c~u]0 = cu~0
(3) [f (t)~u]0 = f 0 (t)~u + f (t)u~0
(4) [~u ~v ]0 = u~0 ~v + ~u v~0
(5) [~u ~v ]0 = u~0 ~v + ~u v~0
(6) Chain Rule: [~u(f (t))]0 = f 0 (t)u~0 (f (t)).
Example 10.7.8
Let ~r(t) be a vector function with a constant length, say c > 0. Show that
~r(t) r~0 (t) = 0.
Solution.
We have
0 = [~r(t) ~r(t)]0 = r~0 (t) ~r(t) + ~r(t) r~0 (t) = 2~r(t) r~0 (t).
Thus, ~r(t) r~0 (t) = 0 and the two vectors are orthogonal

Integrals
The definite integral of a continuous vector function ~r(t) can be defined in
much the same way as for real-valued functions except that the integral is a
vector. But then we can express the integral of ~r(t) in terms of the integrals
of its component functions f (t), g(t), and h(t) as follows
Z b n
X
~r(t)dt = lim ~r(ti )t
a n
i=1
n
! n
! n
!
X X X
= lim f (ti )t ~i + lim g(ti )t ~j + lim h(ti )t ~k
n n n
i=1 i=1 i=1
Z b  Z b  Z b 
= ~
f (t)dt i + ~
g(t)dt j + h(t)dt ~k.
a a a

5
~
We say that R(t) ~ 0 (t) = ~r(t). All antideriva-
is an antiderivative of ~r(t) if R
tives of ~r are then given by the indefinite integral
Z
~ +C
~r(t)dt = R(t) ~

where C~ is a vector constant of integration.


We can extend the Fundamental Theorem of Calculus to continuous vector
functions as follows: Z b
~
~r(t)dt = R(b) ~
R(a).
a

Example 10.7.9
Find ~r(t)dt where ~r(t) = cos t~i + sin t~j + t~k.
R

Solution.
We have
t2
Z Z  Z  Z 
~r(t)dt = cos tdt ~i+ sin tdt ~j+ tdt ~k = sin t~icos t~j+ ~k+C
~
2

6
Arkansas Tech University
MATH 2934: Calculus III
Dr. Marcel B Finan

10.8 Arc Length and Curvature


Let C be a space curve generated by a vector function ~r(t) = f (t)~i + g(t)~j +
h(t)~k such that f 0 (t), g 0 (t), and h0 (t) are continuous. Furthermore, we assume
that the curve is traversed once as t increases. That is, the curve does not
double itself. We want to find the length of the curve between two points
~r(a) and ~r(b). To do this, we start by partitioning the interval [a, b] into n
subintervals by means of the points a = t0 < t1 < t2 < < tn1 < tn = b
in such a way that the portion of the curve between ~r(ti1 ) and ~r(ti ) is
approximated by the vector ~r(ti ) ~r(ti1 ). See Figure 10.8.1.

Figure 10.8.1

In this case, we can measure the length of the line segment and find
p
||~r(ti ) ~r(ti1 )|| = (f (ti ) f (ti1 ))2 + (g(ti ) g(ti1 ))2 + (h(ti ) h(ti1 ))2
s 2  2  2
f (ti ) f (ti1 ) g(ti ) g(ti1 ) h(ti ) h(ti1 )
= + + (ti ti1 )
ti ti1 ti ti1 ti ti1
p
[f 0 (ti )]2 + [g 0 (ti )]2 + [h0 (ti )]2 (ti ti1 ).

Adding the n pieces to obtain


n
X p
arclength(C) [f 0 (ti )]2 + [g 0 (ti )]2 + [h0 (ti )]2 (ti ti1 ).
i=1

Letting n , we obtain
Z bp Z b
arclength(C) = [f 0 (t)]2 + [g 0 (t)]2 + [h0 (t)]2 dt = ||r~0 (t)||dt.
a a

1
This should make sense physically: ~r(t) is the position of a particle moving
in space, r~0 (t) is its velocity, ||r~0 (t)|| is its speed, and to find the total distance
the particle has traveled from time a to time b, we should integrate the speed
from a to b. This should also explain why we asked that the curve not double
back on itself: if it did so, we would be double counting.

Example 10.8.1
Find the length of one turn of the helix ~r(t) =< cos t, sin t, t > .

Solution.
We have
Z 2 p Z 2
arclength = ( sin t)2 + (cos t)2 + 1dt = 2dt = 2 2
0 0

Arc length parameterization


A space curve can have two different parameterizations. For example, ~r(t) =<
t, t2 , t3 >, 1 t 2 and ~u(t) =< et , e2t , e3t >, 0 t ln 2 represent the
same curve. Even though the two parameterizations look different, the for-
mula for the arc length gives the same answer. Indeed,
Z ln 2 Z ln 2
||u~ (t)||dt =
0 e2t + 4e4t + 9e6t dt
0 0
Z ln 2
= et 1 + 4e2t + 9e4t dt
Z0 2 Z 2
= 2 4
1 + 4t + 9t dt = ||r~0 (t)||dt.
1 1

Hence, arc length does not depend on curve parameterization.


Because of the independence of arc length on curve parameterization, it is
useful to express a vector function in terms of arc length. Suppose that C
is a space curve given by a vector function ~r(t) for a t b where r~0 (t) is
continuous and C is traversed once. Suppose we want to parameterize the
curve in terms of the distance traveled from the fixed endpoint ~r(a). To do
this, we define the arc length function by
Z t
s(t) = ||r~0 (u)||du.
a

2
Then by the Fundamental Theorem of Calculus, we have
ds
= ||r~0 (t)||
dt
which is the speed of the object moving on the curve C. If we have a vector
function ~r(t), to convert it to a vector function in terms of arc length we
compute s = s(t) and from this expression we solve for t in terms of s, say
t = t(s), and then we substitute this back into ~r(t) to obtain ~r(t(s)). Thus,
~r(t(s)) is the position vector of the point s units of length along the curve
from its starting point.

Example 10.8.2
Reparameterize the helix ~r(t) =< cos t, sin t, t > with respect to arc length
measured from (1, 0, 0) in the direction of increased t.

Solution.
First, notice that the point (1, 0, 0) corresponds to t = 0. Finding the arc
length function, we obtain
Z t Z t
s = s(t) = ~0
||r (u)||du = 2du = 2t.
0 0

Solving this equation for t, we find t = s .


Hence, the required reparameter-
2
ization is    
s s s
~r(t(s)) =< cos , sin , >
2 2 2

Curvature
Consider a car driving along a curvy road. The tighter the curve, the more
difficult the driving is. Curvature describes this tightness. If the cur-
vature is zero then the curve looks like a line near this point. While if the
curvature is a large number, then the curve has a sharp bend.
In order to discuss curvature, we first start with the following definition: A
space curve C, parameterized by a vector function ~r(t) on an interval I, is
called smooth if r~0 (t) is continuous and different from ~0 in the interval I.
Geometrically, a smooth curve has no sharp corners.
Now, the unit tangent vector

r~0 (t)
T~ (t) =
||r~0 (t)||

3
describes the direction of the curve. Curvature is the measure of how quickly
the unit tangent vector changes direction at a given point on the curve. For
example, the unit tangent vector changes directions quickly when a curve
bends or twists sharply as shown in Figure 10.8.

Figure 10.8.2

Since the definition of the unit tangent vector depends on the parametrization
of ~r, we can make this vector independent of parameterization if we work only
with a parameterization which is intrinsic to the curve. That is, we should
differentiate not with respect to time, but with respect to arc length (or,
equivalently, reparameterize the curve with respect to arc length and then
differentiate with respect to the new parameter). So we define the curvature
of a smooth curve to be the number

dT~
= .

ds

Notice that by applying the Chain Rule, we obtain


dT~
dT~ dT~ ds dT~ dt
= = = ds
.
dt ds dt ds dt

Hence,
||T~ 0 (t)||
= .
||r~0 (t)||

Example 10.8.3
Find the curvature of ~r(t) =< cos t, sin t, t > at t = 4 .

4
Solution.
We have
r~0 (t) = < sin t, cos t, 1 >

||r~0 (t)|| = 2
cos t sin t t
T~ (t) = < , , >
2 2 2
sin t cos t 1
T~ 0 (t) = < , , >
2 2 2
~ 0
||T (/4)|| 1
= =
||r~ (/4)||
0 2
The formula for is more practical to use, but still cumbersome. The formula
given by the following theorem is often more convenient to apply.
Theorem 10.8.1
The curvature of the curve given by the vector function ~r(t) is
||r~0 (t) r~00 (t)||
= .
||r~0 (t)||3
Proof.
From the definition of T~ (t), we have r~0 (t) = ||r~0 (t)||T~ (t) = ds ~
dt
T (t). Taking
the derivative of both sides, we find
d2 s ds
r~00 (t) = 2 T~ (t) + T~ 0 (t).
dt dt
Thus,
   2 
ds ~ d s~ ds ~ 0
r~0 (t) r~00 (t) = T (t) T (t) + T (t)
dt dt2 dt
   2     
ds ~ d s~ ds ~ ds ~ 0
= T (t) T (t) + T (t) T (t)
dt dt2 dt dt
  2   2
ds ds ~ ~ ds
= 2
T (t) T (t) + T~ (t) T~ 0 (t)
dt dt dt
 2
ds
= T~ (t) T~ 0 (t)
dt
=||r~0 (t)||2 T~ (t) T~ 0 (t).

5
Since ||T~ (t)|| = 1, the vectors T~ (t) and T~ 0 (t) are orthogonal (Section 10.7).
Hence, ||T~ (t) T~ 0 (t)|| = ||T~ (t)||||T~ 0 (t)|| = ||T~ 0 (t)||. Thus,

||r~0 (t) r~00 (t)|| = ||r~0 (t)||2 ||T~ 0 (t)||

and
||r~0 (t) r~00 (t)||
=
||r~0 (t)||3
From this formula, we see that the curvature of a straight line (i.e., r~00 (t) = ~0)
is always zero.

Example 10.8.4
Find the curvature of ~r(t) =< t, t2 , t3 > at any point t.

Solution.
We have

r~0 (t) = < 1, 2t, 3t2 >


r~00 (t) = < 0, 2, 6t >
r~0 (t) r~00 (t) = < 6t2 , 6t, 2 >

36t4 + 36t2 + 4
(t) = 3
(1 + 4t2 + 9t4 ) 2

Example 10.8.5
Find the curvature of the parabola y = x2 at any point x.

Solution.
We have

~r(x) = < x, x2 , 0 >


r~0 (x) = < 1, 2x, 0 >
r~00 (x) = < 0, 2, 0 >
r~0 (x) r~00 (x) = < 0, 0, 2 >
2
(x) = 3
(1 + 4x2 ) 2

6
The Normal and Binormal Vectors
We define the unit normal vector to the unit tangent vector T~ to be
the vector orthogonal to T~ and pointing in the direction where the curve is
bending. See Figure 10.8.3.

Figure 10.8.3

Since ||T~ || = 1, we have 0 = (||T~ ||2 )0 = 2T~ T~ 0 and hence T~ T~ 0 = 0 and T~ 0


is orthogonal to T~ . Thus, the unit normal vector is
~0
~ (t) = T (t) .
N
||T~ 0 (t)||

We define the binormal vector to be the unit vector


~
B(t) = T~ (t) N
~ (t).

Example 10.8.6
Find the unit normal and binormal vectors for the circular helix ~r(t) =<
cos t, sin t, t > .

Solution.
We have

r~0 (t) = < sin t, cos t, 1 >



||r~0 (t)|| = 2
r~0 (t) sin t cos t 1
T~ (t) = =< , , >
||r~0 (t)|| 2 2 2

7
cos t sin t
T~ 0 (t) = < , , 0 >
2 2
1
||T~ 0 (t)|| =
2
~0
N ~ (t) = T (t) =< cos t, sin t, 0 >
||T~ 0 (t)||
~i ~j ~k

~ =T~ (t) N ~ (t) = sin
t cos

B(t) t 1
2 2 2
cos t sin t 0
sin t cos t 1
= < , , >
2 2 2

The plane determined by the normal and binormal vectors N ~ (t) and B(t)
~ at
a point P on a curve C is called the normal plane of C at P. It consists of
all lines orthogonal to the tangent vector T~ at P. The plane determined by
the normal and tangent vectors N ~ (t) and T~ (t) at a point P on a curve C is
called the osculating plane of C at P.

8
Arkansas Tech University
MATH 2934: Calculus III
Dr. Marcel B Finan

10.9 Motion in Space: Velocity and Acceleration


In this section we discuss how to find the velocity and acceleration vectors
from the parametric equations of the motion of the object.
Let ~r(t) be the position vector at time t and ~r(t + t) the position at time
t + t. Then the displacement vector between these two positions is ~r(t) =
~r(t + t) ~r(t) as shown in Figure 10.9.1. The average velocity over the
interval [t, t + t] is defined by
~
r(t)
Average velocity = t

and the instantaneous velocity is

~r(t) ~r(t + t) ~r(t)


~v (t) = r~0 (t) = lim = lim
t0 t t0 t
whenever the limit exists. Note that ~v (t) is tangent to the objects path of
motion.

Figure 10.9.1

We define the speed of the object to be the length of the vector ~v (t).
In a similar way, we define the acceleration vector to be

~v (t) ~v (t + t) ~v (t)
~a(t) = lim = lim
t0 t t0 t
whenever the limit exists. We write ~a(t) = ~v 0 (t) = ~r 00 (t)

Example 10.9.1
Find the velocity, speed and acceleration of the motion with position function
~r(t) =< 3 cos (t2 ), 3 sin (t2 ), t2 > .

Solution.
The velocity vector is given by

~v (t) = x0 (t)~i + y 0 (t)~j + z 0 (t)~k = 6t sin (t2 )~i + 6t cos (t2 )~j + 2t~k.

1
The speed is
p
||~v (t)|| = (6t sin t2 )2 + (6t cos t2 )2 + 4t2 = 40t2 = 2t 10.

The acceleration is the vector

~a(t) =x00 (t)~i + y 00 (t)~j + z 00 (t)~k


= 6(sin (t2 ) + 2t2 cos (t2 ))~i + 6(cos (t2 ) 2t2 sin (t2 ))~j + 2~k

Vector integrals can be used to find position vectors when velocity or accel-
eration vectors are known, as in the next example.

Example 10.9.2
Given the acceleration ~a(t) = ~i + t~kwith initial velocity ~v (0) = 5~j and initial
position ~r(0) = ~i, find the velocity and position vector functions.

Solution.
Since ~a(t) = v~0 (t), we have

t2
Z
~v (t) = (~i + t~k)dt = t~i + ~k + C.
~
2
~ Hence,
But ~v (0) = 5~j = C.

t2
Z
~v (t) = (~i + t~k)dt = t~i + 5~j + ~k.
2

Now, since ~v (t) = r~0 (t), we have

t2 t2 t3
Z
~r(t) = (t~i + 5~j + ~k)dt = ~i + 5t~j + + D.
~
2 2 6
~ = ~i. Hence,
But ~r(0) = D

t2 t3
~r(t) = ( + 1)~i + 5t~j +
2 6

Uniform Circular Motion


A circular motion is the motion of an object along a path of radius R as

2
shown in Figure 10.9.2.

Figure 10.9.2

There are two ways to describe the motion of the objectlinear and angular
speed. The linear speed v of the object is the rate at which the distance
traveled is changing. It is defined by the formula
s
v=
t
where s is the distance traveled on the circle.
The angular speed is the rate at which the central angle is changing. It is
given by

= .
t
Its units are radians per seconds.
Thus, a parameterization of the motion is given by

~r(t) = R cos ~i + R sin ~j = R cos (t)~i + R sin (t)~j

where t is the time in seconds.


Now, since ||~r(t)|| = R, we have ~r(t) r~0 (t) = 0 so that the linear velocity of
the object is always tangent to the circle.
The velocity is
~v (t) = R sin (t)~i + R cos (t)~j.
The speed of the object is
p
||~v (t)|| = (R sin (t))2 + (R cos (t))2 = R.

3
Hence, the speed of the object is constant and thus the name uniform.
Next, the acceleration vector is given by

~a(t) = ~v 0 (t) = 2 (R cos (t)~i + R sin (t)~j) = 2~r(t).

Since the acceleration is a multiple of the position, these vectors are paral-
lel. Note the minus sign in the above expression means that the acceleration
points in the opposite direction of ~r(t) that is towards the center of the circle.
Since ~v is perpendicular to ~r, the acceleration is also perpendicular to ~v .

Force
Force is yet another example of a vector quantity. In order to move an object,
something must push on it. The strength of the push is the magnitude of
the force applied. However the direction of the push is important. If you wish
to move a chair in a room, its much more effective to push on it sideways
then to push down on it! In both cases, the same magnitude of force may
be applied to the chair, but the results will be substantially different. Hence
force has direction as well as magnitude and is therefore a vector.
The vector version of Newtons second law of motion states that if, at
any time t, a force F~ (t) acts on an object of mass m producing an acceleration
~a(t) then
F~ (t) = m~a(t).
In the case of uniform circular motion, we have F~ (t) = m 2~r(t) which
shows that the force acts in the direction opposite to the radius vector and
therefore points toward the origin.

Example 10.9.3
A projectile of mass m = 50kg is fired with an initial speed of 500 m/s and
angle of elevation 30 degrees. Assuming air resistance is negligible (so the
only external force is due to gravity), find the following:
(a) the range of the projectile,
(b) the maximum height reached,
(c) the speed at impact.

Solution.
We set up the axes so that the projectile starts at the origin. The initial
velocity vector is ~v (0) = 500 cos 30 i + 500 sin 30 j = 250 3~i + 250~j.
~ ~

The only force acting on the projective is the force of gravity that is pointing

4
downward so that F~ (t) = m~a(t) = mg~j, where g = 9.8 m/s2 . Hence,
~a(t) = g~j. It follows that
~
~v (t) = gt~j + C

~ = ~v (0) = 250 3~i + 250~j. Hence,
and C

~v (t) = 250 3~i + (250 9.8t)~j.
The vector position of the projectile is
Z
~
~r(t) = (250 3~i + (250 9.8t)~j)dt = 250 3t~i + (250t 4.9t2 )~j + D.

Since the projectile starts at the origin, we have D~ = ~r(0) = ~0. Hence,

~r(t) = 250 3t~i + (250t 4.9t2 )~j.
2 x

x
2
Letting x = 250 3t and y = 250t 4.9t we find y = 3 4.9 250 3 =

x 4.9
187,500 x2 which is a parabola in the xyplane.
3
(a) The range of the projectile is the horizontal distance it will travel. To
calculate this, we need to know when it hits the ground. This occurs when
250t 4.9t2 = 0. Solving this equationwe find t = 0 and t = 51 seconds.
Hence, the range of the projectile is 250 3(51) = 22, 083 meters or about 22
km.
(b) The maximum height is the maximum of the function y = 250t 4.9t2
which occurs at the vertex of this parabola, i.e., at t = 1254.9
25.5 and
y 250(25.5) 4.9(25.5)2 3, 189m. q
(c) The speed at the impact is ||~v (51)|| = (250 3)2 + (250 9.81(51))2
500 m/s

Tangential and normal components of acceleration


When studying the motion of an object, it is often useful to resolve the ac-
celeration vector into two componentsone in the direction of the tangent
(the direction we are traveling) and the normal (the direction the curve is
turning). If v(t) = ||~v (t)|| is the speed of an object whose position is given
by the vector function ~r(t), then

r~0 (t) ~v (t)


T~ (t) = =
||r~ (t)||
0 v(t)

5
or
~v (t) = T~ (t)v(t).
Differentiating this equation with respect to t, we find
~a(t) = v 0 (t)T~ (t) + v(t)T~ 0 (t).
Using the expression of the curvature
||T~ 0 (t)||
=
||r~0 (t)||

we find ||T~ 0 (t)|| = v(t). On the other hand, we have


~0 ~0
~ (t) = T (t) = T (t) = T~ 0 (t) = v(t)N
N ~ (t).
~ 0
||T (t)|| v(t)

Hence, ~a(t) can be resolved in terms of the normal and tangent vectors T~ (t)
~ (t) as
and N
~a(t) = v 0 (t)T~ (t) + v 2 (t)N
~ (t).
Letting, aT = v 0 (t) be the tangential component and aN = v 2 (t) be the
normal component, the resolution of ~a(t) is described geometrically in Figure
10.9.3.

Figure 10.9.3
Next, we express aT = v 0 (t) and aN = v 2 t in terms of ~r(t), r~0 (t), and r~00 (t).
We have
~v ~a(t) =v(t)T~ (t) (v 0 T~ (t) + v 2 (t)N
~ (t))
=v(t)v 0 (t)T~ T~ + v 3 (t)T~ N~
=v(t)v 0 (t)||T~ ||2 = v(t)v 0 (t).

6
~a(t) ~0 ~00
Hence, aT = v 0 (t) = ~vv(t) = r ||(t) r (t)
r~0 (t)||
.
On the other hand, using the formula

||r~0 (t) r~00 (t)||


=
||r~0 (t)||3
we arrive at
||r~0 (t) r~00 (t)||
aN = v 2 (t) = .
||r~0 (t)||

Example 10.9.4
Find the tangential and normal components of the acceleration vector of
~r(t) = (3t t3 )~i + 3t2~j.

Solution.
Straight forward calculation, we find

r~0 (t) =(3 3t2 )~i + 6t~j


p
||r~0 (t)|| = (3 3t2 )2 + 36t2 = 9 + 18t2 + 9t4
p
= 9(1 + t2 )2 = 3(1 + t2 )
r~00 (t) = 6t~i + 6~j
r~0 (t) r~00 (t) =(18t2 + 18)~k
r~0 (t) r~00 (t) =18t3 + 18t
r~0 (t) r~00 (t) 18t3 + 18t
aT = = = 6t
||r~0 (t)|| 3 + 3t2
||r~0 (t) r~00 (t)|| 18t2 + 18
aN = = =6
||r~0 (t)|| 3(1 + t2 )

7
Arkansas Tech University
MATH 2934: Calculus III
Dr. Marcel B Finan

11.1 Functions of Two/Three Variables


In many real world problems one encounters a quantity that depends on more
than one input. For example, if an amount of money A is invested at a simple
annual interest rate r for a period of t years then the balance at the end of t
years is given by the formula B = A(1 + r)t . Thus, B can be regarded as a
function of three variables A, r, and t. In function notation we will write

B = f (A, r, t).

Multivariable calculus is the study of functions of more than one variable.


In this course we will mainly focus on functions having two or three variables.
However, functions of four, five, or more variables do occur in models of the
physical world and the results presented in the course also apply to such func-
tions.
First, we introduce the definition of a function of two variables: A scalar-valued
function of two real variables x and y is a rule, f, that associates with each
choice of x and y a single real number f (x, y) called the value of f at (x, y). In
function notation, we write
z = f (x, y).
We call z the dependent variable and x and y the independent variables.
The set Dom(f ) = {(x, y) : f (x, y) exists} is called the domain of f. The range
of f is the set of all possible values of f (x, y) for each (x, y) in the domain of f.

Example 11.1.1 p
Consider the function f (x, y) = 1 x2 y 2 .
(a) Find f ( 21 , 12 ) and f (2, 1).
(b) Find the domain and the range of the function f.

Solution. q q
(a) We have f ( 12 , 12 ) = 1 14 14 = 12 . But f (2, 1) = 1 4 1 = 4
which is an imaginary number. This means that (2, 1) is not in the domain of
f.
(b) The domain consists of all points (x, y) that satisfy the inequality x2 +y 2 1.
That is, the domain is the disk centered at the origin and with radius 1. The
range is the closed interval 0 z 1

Example 11.1.2
Find the domain and range of the function f (x, y) = 1 .
xy

Solution.
The domain is {(x, y)|y < x} because of the square root in the denominator and
the range is {z|z > 0}

1
Example 11.1.3 q
Find the domain and range of the function f (x, y) = xy .

Solution.
The domain is {(x, y)|xy 0 and y 6= 0} and the range is {z|z 0}

Graph of z = f (x, y)
If you recall that the graph of a function f of one variable x is the set of all
points (x, y) in the two dimensional plane such that y = f (x) and x in the do-
main of f. That is, the graph is a curve in the 2-D system. In a similar way, the
graph of a function f of two variables x and y is the set of all ordered triples
(x, y, z) such that z = f (x, y) and (x, y) is in the domain of f. The graph is a
surface in the 3-D space.

Example 11.1.4
Sketch the graph of z = f (x, y) = 12 3x 4y.

Solution.
The graph is a plane shown in Figure 11.1.1. A function of the form z =
f (x, y) = c + ax + by is called a linear function. The graph is a plane in the
3-D space. To graph the plane one usually finds the intersection points with the
three axes and then graphs the triangle that connects those three points. This
triangle will be a portion of the plane and it will give us a fairly decent idea on
what the plane itself should look like

Figure 11.1.1

Example 11.1.5 p
Sketch the graph of z = f (x, y) = 1 x2 y 2 .

Solution.p
From z = 1 x2 y 2 we obtain x2 + y 2 + z 2 = 1. Thus, the graph of f is the
upper half of the sphere centered at the origin and with radius 1 as shown in

2
Figure 11.1.2

Figure 11.1.2

Example 11.1.6
Sketch the graph of z = f (x, y) = x2 + y 2 .

Solution.
The graph is the paraboloid shown in Figure 11.1.3

Figure 11.1.3

Example 11.1.7
Sketch the graph of z = f (x, y) = 6 x2 y 2 .

3
Solution.
First note that z = 6 (x2 + y 2 ). The graph of this function is a reflection of
the previous paraboloid about the xyplane followed by a vertical translation
6 units up along the zaxis as shown in Figure 11.1.4 .

Figure 11.1.4
As you can see, graphing functions in space manually is quite difficult and is not
an easy matter. You will not need to do this. We usually graph function of two
variables using a graphical device such as a computer or a graphing calculator.

Example 11.1.8
Use a computer or a graphing calculator to graph the cyclinder z = f (x, y) = x2 .

Solution.
Note that the missing variable is y. So the cylinder has axis of symmetry the
y-axis. The graph is shown in Figure 11.1.5

Figure 11.1.5
Level Curves and Contour Diagrams
Graphs provide one way of visualizing functions of two variables. Another im-
portant way of visualizing such functions is by drawing their contour diagrams.
Given a function of two variables z = f (x, y). The cross-section between the

4
surface and a horizontal plane projected onto the xyplane is called a level
curve or a contour curve. Thus, level curves have algebraic equations of the
form f (x, y) = k for all possible values of k. A contour diagram or contour
map of a function f (x, y) is a 2-dimensional graph showing several level curves
in the xyplane corresponding to several values of k.

Example 11.1.9 p
Draw a contour diagram of z = f (x, y) = x2 + y 2 showing several level curves.

Solution.
The surface representing the given function is a cone centered at the origin as
shown in Figure 11.1.6(a). Horizontal planes crossing this surface trace circles
in these planes. Thus, the level curves are circles centered at the origin in the
xyplane. Figure 11.1.6(b) shows a contour map consisting of the level curves
k = 0, 1, 2, 3, 4, 5

Figure 11.1.6

Remark 11.1.1
One can create a contour diagram from a surface and vice versa. If the surface
is given, then we create contour diagrams by joining all the points at the same
height on the surface and dropping the curve into the xy plane. On the other
hand, if the contour diagram is given with a constant value assigned to each
contour curve, we obtain the surface by lifting each contour curve to a height
equal to its assigned value

Example 11.1.10
Find equations for the level curves of f (x, y) = x2 y 2 and draw a contour
diagram for f.

Solution.
The level curves are the curves of the form x2 y 2 = k, for all possible values of
k. For k = 0 we find the two perpendicular lines y = x and y = x. For k 6= 0

5
the graphs of x2 y 2 = k are hyperbolas with asymptotes consisting of the lines
y = x. Figure 11.1.7 shows the contour map. Note that the x2 y 2 > 0 is a
hyperbola that crosses the xaxis whereas x2 y 2 < 0 is a hyperbola crossing
the yaxis

Figure 11.1.7

Example 11.1.11
Draw a contour diagram of the function f (x, y) = 2x + 3y + 6 showing several
level curves.

Solution.
The contour diagram is a set of evenly spaced parallel lines of common slope
23 as shown in Figure 11.1.8

Figure 11.1.8

Functions of Three Variables


Functions of three variables appear in many applications. For instance, the
temperature T at a point on the surface of the earth depends on the longitude x
and the latitude y of the point and on the time t, so we could write T = f (x, y, t)
so that T is a function of three variables.
Functions of three variables are defined in the same way as functions of two
variables. We say that f is a function of the variables x, y, z if f is a rule that
assigns to every ordered triples (x, y, z) a unique number w = f (x, y, z). The
domain of f is the set of all ordered triples (x, y, z) such that f (x, y, z) exists.

6
Thus, the domain is a subset of 3-D. The range of f is the collection of all
numbers f (x, y, z) where (x, y, z) is in the domain of f.

Example 11.1.12 p
Find the domain and range of the function w = f (x, y, z) = 1 x2 y 2 z 2 .

Solution.
The domain consists of all ordered triples (x, y, z) satisfying the inequality
x2 + y 2 + z 2 1. Thus, the domain is the unit ball centered at the origin
and with radius 1. The range is the interval 0 w 1

The graph of f is the set of the 4-D space consisting of all points of the form
(x, y, z, f (x, y, z)).
We have seen that a function of a single variable has a graph that represents a
curve in the xyplane, i.e., a two dimensional space or 2-D space. Likewise, the
graph of a function of two variables is a surface in 3-D. Going to a function of
three variables gives us a surface in 4-D space which cant be drawn. This makes
visualizing functions with three or more variables much more difficult. For func-
tions of three variables however, their contours are surfaces in 3-D space, so at
least we can visualize them by graphing these surfaces.
By a level surface of a function w = f (x, y, z) we mean a 3-D surface of the
form f (x, y, z) = c, where c is a constant. The function f can be represented by
the family of level surfaces obtained by allowing c to vary. Keep in mind that
the value of the function is constant on each level surface.

Example 11.1.13
What do the level surfaces of f (x, y, z) = x2 + y 2 + z 2 look like?

Solution.
The level surfaces are given by the equations x2 + y 2 + z 2 = c where c 0.
The set of level surfaces for this function are just a concentric set of spheres of
different radii. Figure 11.1.9 shows the inside of the lower half spheres

Figure 11.1.9

7
Example 11.1.14
What do the level surfaces of f (x, y, z) = x2 + y 2 look like?

Solution.
Level surfaces have equations of the form

x2 + y 2 = c

where c 0. Each level surface is a circular cylinder of radius c around the
zaxis. The level surfaces are concentric cylinders as shown in Figure 11.1.10.
Note that f has smaller values on the narrow cylinders near the zaxis and
larger values on the wider ones

Figure 11.1.10

Example 11.1.15
What do the level surfaces of f (x, y, z) = x2 + y 2 z 2 look like?

Solution.
Level surfaces are given by the equation x2 + y 2 z 2 = c. For c < 0 the level
surface is a surface known as a hyperboloid with two sheets. If c = 0 we
obtain a cone. If c > 0 the surface is called a hyperboloid with one sheet.
See Figure 11.1.11

Finally we note that the graph of a two-variable function z = f (x, y) can be


thought of as one member in a family of level surfaces representing the three-
variable function:
g(x, y, z) = f (x, y) z
Indeed, the graph of z = f (x, y) is just the level surface g(x, y, z) = 0. Con-
versely, a single level surface g(x, y, z) = c can be regarded as the graph of

8
a function f (x, y) if it is possible to solve for z in terms of x and y. For ex-
ample, the level surface x2 + y 2 z + 3 = 0 is just the graph of the function
z = f (x, y) = x2 + y 2 + 3.

Figure 11.1.11

9
Arkansas Tech University
MATH 2934: Calculus III
Dr. Marcel B Finan

11.2 Limits and Continuity of Functions of Two Variables


In this section, we present a formal discussion of the concept of continuity of
functions of two variables. Our discussion is not limited to functions of two
variables, that is, our results extend to functions of three or more variables.
The definition of continuity requires a discussion of the concept of limits. Before
we introduce this concept for functions in two variables, let us recall the one
dimensional version:
We say that a function f (x) has a limit L at a if and only if for every  > 0 there
exists a positive number depending on  such that for any x in the domain of
f with the property 0 < |x a| < we have |f (x) L| < . In symbol, we write

lim f (x) = L
xa

or f (x) L as x a.
Geometrically, the definition says that for any  > 0 (as small as we want), there
is a > 0 (sufficiently small) such that any point inside the interval (a , a + )
is mapped to a point inside the interval (L , L + ) as shown in Figure 11.2.1.

Figure 11.2.1
A similar definition extends to functions in two variables: We say that L is the
limit of a function f at the point (a, b), written

lim f (x, y) = L
(x,y)(a,b)

if f (x, y) is as close to L as we please whenever the distance from the point


(x, y) to the point (a, b) is sufficiently small, but not zero.
Using  definition we say that L is the limit of f (x, y) as (x, y) approaches

1
(a, b) if and only if for p
every given  > 0 we can find a > 0 such that for any
point (x, y) where 0 < (x a)2 + (y b)2 < we have |f (x, y) L| < .
What does this mean in words? To say that L is the limit of f (x, y) as (x, y)
(a, b) means that for any given  > 0, we can find an open punctured disk (i.e.,
without the center and the boundary) centered at (a, b) such that for any point
(x, y) inside the disk the difference f (x, y) L is within , i.e., L  < f (x, y) <
L + . Figure 11.2.2 illustrates this.

Figure 11.2.2
Example 11.2.1
Let f (x, y) = x2 + y 2 . Is lim(x,y)(1,1) f (x, y) = 3?
Solution.
Let  = 0.1. Is there a > 0 such that all the points (x, y) inside the open disk
with radius and centered at (1, 1) satisfy |x2 + y 2 3| < 0.1 or equivalently
2.9 < x2 + y 2 < 3.1? Clearly, any such open disk will share points with the open
disk centered at (1, 1) and with radius 0.2. But any point (x, y) in this latter
disk satisfies (x 1)2 + (y 1)2 < 0.04 or x2 + y 2 2(x + y) + 2 < 0.04. Since
0.8 < x < 1.2 and 0.8 < y < 1.2 we find 3.2 < 2(x + y) < 4.8. This implies
that f (x, y) = x2 + y 2 < 0.04 2 + 4.8 = 2.84 < 2.9. Hence any point in the
disk centered at (1, 1) and radius 0.2 will fall outside the interval (2.9, 3.1). In
particular, this is true for any point in the disk centered at (1, 1) and radius .
We conclude that lim(x,y)(1,1) f (x, y) 6= 3

As in the case of functions of one variable, limits of functions of two variables


possess the following properties:
The limit, if it exists, is unique.
The limit of a sum, difference, product, is the sum, difference, product of
limits.
The limit of a quotient is the quotient of limits provided that the limit in the
denominator is not zero.
Remark 11.2.1
In the case of functions of one variable, if a function f (x) has a limit L at

2
x = a then the limit of f (x) as x approaches a from either the left or right must
be L. Similar situation occurs for a function f (x, y) of two variables with the
difference that the point (x, y) can approach (a, b) in infinite directions. Hence,
if you can find two directions toward (a, b) with two different limits then the
function has no limit as (x, y) (a, b).

We can now define what we mean by continuity in terms of limit. Intuitively,


we expect our definition to support the idea that there are no breaks or gaps
in the function if it is continuous. The continuity of functions of two variables
is defined in the same way as for functions of one variable:
A function f (x, y) is continuous at the point (a, b) if the following two condi-
tions are satisfied:
(a) Both f (a, b) and lim(x,y)(a,b) f (x, y) exist;
(b) lim(x,y)(a,b) f (x, y) = f (a, b).

A function is continuous on a region R in the xyplane if it is continu-


ous at each point in R. A function that is not continuous at (a, b) is said to be
discontinuous at (a, b).
Since the condition lim(x,y)(a,b) f (x, y) = f (a, b) means that f (x, y) is close
to f (a, b) when (x, y) is close to (a, b) we see that our definition does indeed
correspond to our intuitive notion that the graph of f (x, y) has no breaks or
gaps around (a, b).
Like the case of functions of one variable, it can be shown that sums, prod-
ucts, quotients (where denominator function is not zero), and compositions of
continuous functions are also continuous.

Example 11.2.2
Show that (
x2 y
x2 +y 2 (x, y) 6= (0, 0)
f (x, y) =
0 (x, y) = (0, 0)
is continuous at (0, 0).

Solution.
This function is clearly continuous everywhere except at (possibly) (0, 0). Lets
check continuity
p at (0, 0). Let  > 0 be given. Can we find a > 0 such that
if 0 < x + y < then |f (x, y) 0| < ? Let = 2 and suppose
2 2
that
p
2 2 2 2 2 x2
0 < x + y < . Then, using the fact that x x + y , i.e. x2 +y2 1 we
have
x2 y x2

|y| |y| x2 + y 2 < =  < .
p
|f (x, y) 0| = 2 2
=
2 2
x +y x +y 2

Hence, f (x, y) is continuous at (0, 0)

Remark 11.2.2
According to the definition of continuity, f (x, y) is discontinuous at (a, b) if and

3
only if either f (a, b) does not exist or lim(x,y)(a,b) f (x, y) does not exist or
lim(x,y)(a,b) f (x, y) 6= f (a, b).

Example 11.2.3
Show that (
x2
x2 +y 2 (x, y) 6= (0, 0)
f (x, y) =
0 (x, y) = (0, 0)
is discontinuous at (0, 0).

Solution.
Again this function is clearly continuous everywhere except (possibly) at (0, 0).
Now lets look at the limit of f (x, y) as (x, y) approaches (0, 0) along two different
paths. First, lets approach (0, 0) along the xaxis, i.e., y = 0.

x2
lim f (x, y) = lim = 1.
(x,y)(0,0) (x,0)(0,0) x2+0

Now, lets approach (0, 0) along the yaxis, i.e., x = 0.


0
lim f (x, y) = lim = 0.
(x,y)(0,0) (0,y)(0,0) 0 + y 2

Since the limit is not the same along the two different directions, we conclude
that f (x, y) is discontinuous at (0, 0)

4
Arkansas Tech University
MATH 2934: Calculus III
Dr. Marcel B Finan

11.3 Derivatives of Functions of Two Variables: The Par-


tial Derivative Function
Suppose we have to take the derivative of a function f (x, y). How does this differ
from taking the derivative of a function f (x)?
You recall that the derivative of a function f (x) is defined in terms of limit as
f (x + h) f (x)
f 0 (x) = lim
h0 h
provided the limit exists. The derivative measures the rate at which f (x) is
changing at x. Geometrically, f 0 (x) represents the slope of the tangent line of
f (x) at x as shown in Figure 11.3.1.

Figure 11.3.1
Now, let z = f (x, y) and (a, b) be a point in its domain. The variable z is the
dependent variable since it depends on both x and y. The variables x and y are
the independent variables and they are allowed to vary independently of each
other. Thus, we study the influence of x and y separately on the value of the
function f (x, y) by holding one fixed and letting the other vary. In this case,
one studies how fast the function is changing in either the x direction or the
ydirection.
In the xdirection we hold y fixed by setting y = b. In this case, the function
f (x, b) is a function in the variable x, say f (x, b) = g(x). If g(x) is differentiable
at x = a then we can write
g(a + h) g(a) f (a + h, b) f (a, b)
g 0 (a) = lim = lim .
h0 h h0 h
Alternative notations for g 0 (a) are
z f f (a + h, b) f (a, b)
fx (a, b) = (a, b) = (a, b) = lim .
x x h0 h

1
We call fx (a, b) the partial derivative of f (x, y) with respect to x at (a, b).
Similarly, the partial derivative of f (x, y) with respect to y at (a, b) is defined
by
z f f (a, b + h) f (a, b)
fy (a, b) = (a, b) = (a, b) = lim .
y y h0 h
Partial Derivatives: Geometric Interpretation
Geometrically, f (x, b) is the cross section obtained when we cut the surface
f (x, y) by the vertical plane y = b. The slope of the tangent line to this cross-
section at x = a is the partial derivative fx (a, b) as shown in Figure 11.3.2.
Likewise, the graph obtained by cutting the surface by the vertical plane x = a
has a tangent line at y = b with slope fy (a, b).

Figure 11.3.2
Estimating Partial Derivatives Using Contour Diagrams
Numerical estimates of the partial derivatives of f (x, y) at a point (a, b) can be
easily made from a contour diagram of the function f (x, y). To estimate fx (a, b)
move from the contour curve through (a, b) in the xdirection and make note
of the x required until you hit the next contour curve. Let z be the change
in the z values. Then fx (a, b) can be estimated using a difference quotient,i.e.,

z
fx (a, b) .
x
Likewise, to estimate fy (a, b) move from the contour curve through (a, b) in the
ydirection and make note of the y required until you hit the next contour
curve. Let z be the change in the z values. Then
z
fy (a, b) .
y
Example 11.3.1
Consider the function z = f (x, y) = x2 + y 2 . A contour diagram is given in

2

Figure 11.3.3. Estimate fx ( 2, 0) and fy (0, 2).

Figure 11.3.3

Solution.
Approaching ( 2, 0) from the left, we find
z 21 1
fx ( 2, 0) = = .
x 21 21

Approaching ( 2, 0) from the right, we find
z 23 1
fx ( 2, 0) = = .
x 2 3 3 2
Thus,

 
1 1 1
fx ( 2, 0) + .
2 21 3 2

Likewise, approaching (0, 2) from below, we find
z 21 1
fy (0, 2) = = .
x 21 21

Approaching ( 2, 0) from above, we find
z 23 1
fy (0, 2) = = .
x 2 3 3 2
Thus,

 
1 1 1
fy (0, 2) +
2 21 3 2

3
Estimating Partial Derivatives from Numeric Data
Consider a rectangular plate of length 5 m and width 3 m. For any point (x, y)
in the rectangular plate we let z = T (x, y) represent the temperature (in C) at
(x, y). The following table provides some numeric values of T (x, y).

x/y 0 1 2 3
0 120 125 100 85
1 135 128 110 90
2 155 135 120 110
3 160 160 145 135
4 160 175 190 155
5 150 160 170 180

Let us estimate Tx (2, 1) and Ty (2, 1). We use difference quotients in the estima-
tion. Thus, we can write
 
1 T (2, 1) T (1, 1) T (3, 1) T (2, 1)
Tx (2, 1) +
2 21 32
 
1 135 128 160 135
= + = 16 C/m
2 21 32

A similar computation shows that

Ty (2, 1) 17.5 C/m.

Remark 11.3.1
If w = f (x, y, z) and (a, b, c) is a point in its domain then we define

f f (a + h, b, c) f (a, b, c)
(a, b, c) = lim
x h0 h
f f (a, b + h, c) f (a, b, c)
(a, b, c) = lim
y h0 h
f f (a, b, c + h) f (a, b, c)
(a, b, c) = lim .
z h0 h
Partial derivatives of functions of three or more variables are found by the same
method as functions of two variables: Differentiate with respect to one variable,
regarding the other variables as constants.

Higher Order Partial Derivatives


Just as we had higher order derivatives with functions of one variable we will
also have higher order derivatives of functions of more than one variable.
Consider the case of a function of two variables z = f (x, y), since both of the
first order partial derivatives are also functions of x and y we could in turn
differentiate each with respect to x or y. This means that for the case of a

4
function of two variables there will be a total of four possible second order
derivatives. The second order partial derivatives are

2f
 
f
(fx )x =fxx = =
x x x2
2f
 
f
(fx )y =fxy = =
y x yx
2f
 
f
(fy )x =fyx = =
x y xy
2f
 
f
(fy )y =fyy = =
y y y 2
The partial derivatives fxy and fyx are called mixed partial derivatives.

Example 11.3.2
Find fxx , fyy , fxy and fyx given that f (x, y) = sin (xy).

Solution.
We have

fx (x, y) =y cos (xy)


fxx (x, y) = y 2 sin (xy)
fxy (x, y) = cos (xy) xy sin (xy)
fy (x, y) =x cos (xy)
fyy (x, y) = x2 sin (xy)
fyx (x, y) = cos (xy) xy sin (xy)

Example 11.3.3
Find fxx , fyy , fxy and fyx given that f (x, y) = x3 + 2xy.

Solution.
We have

fx (x, y) =3x2 + 2y
fxx (x, y) =6x
fxy (x, y) =2
fy (x, y) =2x
fyy (x, y) =0
fyx (x, y) =2

Observe that in the last two examples of this section the mixed partials fxy and
fyx are equal, i.e. fxy = fyx . This is a general result as given by the following
theorem.

5
Theorem 11.3.1
If the mixed partial derivatives fxy and fyx are continuous at a point (a, b) then

fxy (a, b) = fyx (a, b).

Proof.
For h 6= 0 small we define the function

F (h) = [f (a + h, b + h) f (a + h, b)] [f (a, b + h) f (a, b)].

If we let g(x) = f (x, b + h) f (x, b) then the previous equality can be expressed
in terms of g as follows

F (h) = g(a + h) g(a).

By the Mean Value Theorem, there is a number c between a and a + h such


that
g(a + h) g(a) = g 0 (c)h = h[fx (c, b + h) fx (c, b)].
Applying the Mean Value Theorem again, this time to fx we get a number d
between b and b + h such that

fx (c, b + h) fx (c, b) = fxy (c, d)h.

Combining these equations, we obtain

F (h) = h2 fxy (c, d).

If h 0 then (c, d) (a, b), so the continuity of fxy at (a, b) gives

F (h)
lim = lim fxy (c, d) = fxy (a, b).
h0 h2 (c,d)(a,b)

Similarly, by writing

F (h) = [f (a + h, b + h) f (a, b + h)] [f (a + h, b) f (a, b)]

and using the Mean Value Theorem twice and the continuity of fyx at (a, b) we
obtain
F (h)
lim = lim fyx (c, d) = fyx (a, b).
h0 h2 (c,d)(a,b)

It follows that fxy (a, b) = fyx (a, b) as desired

6
Arkansas Tech University
MATH 2934: Calculus III
Dr. Marcel B Finan

11.4 Linear Approximations and Differentials of z = f (x, y)


In this section, we discuss two topics: (1) approximating the value of a func-
tion z = f (x, y) by using a linear function and (2) defining the differential of
z = f (x, y).

Local Linearity of Multivariable Functions


For single variable functions, local linearity is a property of differentiable
functions that roughly says that if you zoom in on a point on the graph of a
differentiable function, the graph will eventually look like a straight line with
a slope equal to the derivative of the function at the point. The advantage of
this property is that function values at nearby points can be estimated using
the tangent line, i.e., a linear function.
The same is true for functions in two variables. A function in two variables is
differentiable at P (a, b) if whenever we zoom in over sufficiently small region
around a point P (a, b) on the surface of the function we see that the surface is
almost a plane containing P (a, b). Thus, a differentiable function of two vari-
ables is well approximated locally by a tangent plane,i.e., a linear function in
the variables x and y. The tangent plane to a surface S at a point P is the plane
that consists of all possible tangent lines at P to curves that lie on S and pass
through P.
What is the equation of the tangent plane? Let z = L(x, y) be the equation of
this plane. Then A(x a) + B(y b) + C(z f (a, b)) = 0 which can be written
as
z f (a, b) = (x a) + (y b).
The intersection of this plane with the plane x = a is the tangent line to the
trace between S and the plane x = a at P. The equation of that tangent line is

z f (a, b) = (y b).

Thus, = fy (a, b). Likewise, = fx (a, b). Hence, from this we conclude that

z = L(x, y) = fx (a, b)(x a) + fy (a, b)(y b) + f (a, b).

Figure 11.4.1 shows the graph of a function with a tangent plane.

Figure 11.4.1

1
Example 11.4.1
Find the equation of the tangent plane to the surface z = 4x3 y 2 + 2y at the
point (1, 2, 12).

Solution.
Since fx (x, y) = 12x2 y 2 and fy (x, y) = 8x3 y + 2, we find fx (1, 2) = 48 and
fy (1, 2) = 14. Also, f (1, 2) = 12. Hence, the equation of the tangent plane
is given by
z = 48(x 1) 14(y + 2) + 12
or
z = 48x 14y 64
Now as we mentioned above, the tangent plane is a good approximation of
f (x, y) locally at the point (a, b). Hence, for points near (a, b) we can write:

f (x, y) fx (a, b)(x a) + fy (a, b)(y b) + f (a, b).

This equation is sometimes referred to as a local linearization (or tangent


plane approximation) of f (x, y) at the point (a, b).

Example 11.4.2
Find the local linearization of f (x, y) = 4x3 y 2 + 2y at the point (1, 2, 12).
Estimate f (0.9, 2.1) using this linearization and compare your answer to the
true value.

Solution.
By the previous example, for points near (1, 2, 12), we have

f (x, y) 48x 14y 64.

Thus,
f (0.9, 2.1) 48(0.9) 14(2.1) 64 = 8.6.
The true value is

f (0.9, 2.1) = 4(0.9)3 (2.1)2 + 2(2.1) = 8.65956

Although we have confined our discussion and examples to functions of two


variables, the above arguments are easily extended to functions having three
or more variables. In particular, for a function of three variables f (x, y, z), the
local linearization at (a, b, c) becomes:

f (x, y, z) fx (a, b, c)(x a) + fy (a, b, c)(y b) + fz (a, b, c)(z c) + f (a, b, c).

The Differential of z = f (x, y)


The formula for the local linearization can be used to estimate the change in
the value of the function locally:

f (x, y) fx (a, b)(x a) + fy (a, b)(y b) + f (a, b) (11.4.1)

2
or
f (x, y) f (a, b) fx (a, b)(x a) + fy (a, b)(y b).
By letting f = f (x, y) f (a, b), x = x a, and y = y b we can write

f fx (a, b)x + fy (a, b)y.

For arbitrary x and y we can introduce the function

df = fx (a, b)dx + fy (a, b)dy.

This function is called the differential of f at (a, b). Note that the differential
of f is a linear function of the variables dx and dy. df represents the change in
f locally due to changes in both x and y.
The geometrical interpretation of df is shown in Figure 11.4.2.

Figure 11.4.2
With the definition of a differential, Equation (11.4.1) can be written f (x, y)
f (a, b) + dz or z dz so that dz is an estimate of the change of the function
values at two different points.

Example 11.4.3
Compute the differential of f (x, y) = 4x3 y 2 + 2y if x changes from 1 to 0.9 and
y changes from 2 to 2.1. Compare the values of z and dz.

Solution.
We have

df = fx (x, y)dx + fy (x, y)dy = 12x2 y 2 dx + (8x3 y + 2)dy.

3
Putting x = 1, y = 2, dx = 0.1 and dy = 0.1, we find

df = 48(0.1) 14(0.1) = 3.2.

Note that the actual change is z = f (0.9, 2.1) f (1, 2) = 8.65956 12 =


3.34044

We conclude this section with the following definition: If a function z = f (x, y)


has continuous first order partial derivatives at a point (a, b) then we say that
f (x, y) is differentiable at (a, b).

Example 11.4.4
y

Show that f (x, y) = y + sin x is differentiable for all (x, y) with x 6= 0.

Solution.
Computing the first order partial derivatives, we find
y y
fx (x, y) = 2 cos
x x
and
1 y
fx (x, y) = 1 + cos .
x x
Since both fx (x, y) and fy (x, y) are continuous for x 6= 0, f (x, y) is differentiable
for (x, y) with x 6= 0

4
Arkansas Tech University
MATH 2934: Calculus III
Dr. Marcel B Finan

11.5 Chain Rule for Functions of Two Variables/Implicit


Differentiation
From single variable calculus we recall that the chain rule is used to differentiate
composite functions of the form f (g(x)). Is there a version of the chain rule for
functions in two variables?
For functions of several variables, composite functions can be generated in a
variety of ways. Consider the following two examples:
Let z = f (x, y), x = x(t), y = y(t) then we can form the composite function
z = f (x(t), y(t)).
Let z = f (x, y), x = x(s, t), y = y(s, t) then we can form the composite
function z = f (x(s, t), y(s, t)).
In the first case, using local linearization of the previous section, we can write

z(t + h) z(t) fx (x, y)(x(t + h) x(t)) + fy (x, y)(y(t + h) y(t)).

Divide through by h 6= 0 to obtain

z(t + h) z(t) x(t + h) x(t) y(t + h) y(t)


fx (x, y) + fy (x, y) .
h h h
As h 0 the above approximation becomes exact obtaining
dz dx dy
= fx (x, y) + fy (x, y) .
dt dt dt
This formula is known as the chain rule.

Example 11.5.1
Compute dz
dt for z = f (x, y) = xe
xy
with x(t) = t2 and y(t) = t1 .

Solution.
Using the formula for the chain rule we find
dz f dx f dy
= +
dt x dt y dt
=(e + xyexy )(2t) + (x2 exy )(t2 )
xy

=2t(exy + xyexy ) t2 x2 exy


=2tet (1 + t) t2 et = tet (2 + t)

Next, we consider a function z = f (x, y) where x = x(s, t) and y = y(s, t). Thus,
z is a function of the variables s and t. What are z z
s and t ?

1
First, let us fix t. Then z = f (x(s, t), y(s, t)) depends on s alone, and so we
apply the chain rule above to write
z z x z y
= + .
s x s y s
Similarly, we can write
z z x z y
= + .
t x t y t
Example 11.5.2
z z
If z = ex sin y, x = st2 , and y = s2 t, find s and t .

Solution.
Applying the chain rule we obtain
z z x z y
= +
s x s y s
=(e sin y)(t2 ) + (ex cos y)(2st)
x

2 2
=t2 est sin (s2 t) + 2stest cos (s2 t).
Similarly,
z z x z y
= +
t x t y t
=(ex sin y)(2st) + (ex cos y)(s2 )
2 2
=2stest sin (s2 t) + s2 est cos (s2 t)
The two versions of the chain rule discussed above can be visualized in Figure
11.5.1.

Figure 11.5.1

Implicit Differentiation
Recall from single variable calculus that the chain rule was useful in finding the
derivative of an implicit function, i.e., a function defined by an equation of the
form F (x, y) = 0 where y is a function of x. The same idea applies for multi-
variable calculus. We illustrate implicit differentiation in the next examples.

2
Example 11.5.3
Let z be a function of x and y defined implicitly by the equation x2 + cos (ez ) +
z z
3y 2 = 5. Find x and y .

Solution.
First, taking the derivative of each term with respect to x, we find
z z z 2x
2x e sin (ez ) = 0 = = z .
x x e sin (ez )

Likewise, taking the derivative of each term with respect to y, we find


z z z 6y
e sin (ez ) + 6y = 0 = = z
y y e sin (ez )

Example 11.5.4
z z
Find x and y if x2 y 2 + z 2 2z = 10.

Solution.
First, taking the derivative of each term with respect to x, we find
z z z x
2x + 2z 2 = 0 = = .
x x x 1z
Similarly, taking the derivative of each term with respect to y, we find
z z z y
2y + 2z 2 = 0 = =
x x y z1

3
Arkansas Tech University
MATH 2934: Calculus III
Dr. Marcel B Finan

11.6 Directional Derivatives and the Gradient Vector


Given a function z = f (x, y) and let (x0 , y0 ) be in the domain of f. We wish
to find the rate of change of f at (x0 , y0 ) in the direction of a unit vector
~u =< a, b > . To do this, we consider the vertical plane to the graph S of f that
passes through the point P (x0 , y0 , z0 ) in the direction of ~u. This plane intersects
the graph S in a curve C. (See Figure 11.6.1.)

Figure 11.6.1
The slope of the tangent line T to C at the point P is the rate of change of z in
the direction of ~u. Let Q(x, y, z) be an arbitrary point on C and let P 0 (x0 , y0 , 0)
and Q0 (x, y, 0) be the orthogonal projection of P and Q respectively onto the

xyplane. Then the vector P 0 Q0 =< x x0 , y y0 > is parallel to ~u so that
00
P Q = h~u for some scalar h. Hence, x = x0 + ha, y = y0 + hb and
f (x, y) f (x0 , y0 ) f (x0 + ha, y0 + hb) f (x0 , y0 )
= .
h h
If we take the limit of the above average rate as h 0, we obtain the rate of
change of z(with respect to distance) in the direction of ~u, which is called the
directional derivative of f at (x0 , y0 ) in the direction of ~u. We write
f (x0 + ah, y0 + bh) f (x0 , y0 )
f~u (x0 , y0 ) = lim .
h0 h
Notice that if ~u = ~i then a = 1 and b = 0 so that f~u (x0 , y0 ) = fx (x0 , y0 ). That
is, fx is the rate of change of f in the x direction. Likewise, if ~u = ~j then
a = 0 and b = 1 so that f~u (x0 , y0 ) = fy (x0 , y0 ).
The following theorem provides a formula for computing the directional deriva-
tive.

1
Theorem 11.6.1
If f is a differentiable function of x and y, then f has a directional derivative in
the direction of any unit vector ~u =< a, b > and

f~u (x, y) = fx (x, y)a + fy (x, y)b.

Proof.
Fix a point (x0 , y0 ) in the domain of f and consider the single variable function
g(h) = f (x0 + ha, y0 + hb). Then

g(h) g(0) f (x0 + ah, y0 + bh) f (x, y)


g 0 (0) = lim = lim = f~u (x0 , y0 ).
h0 h h0 h
Let x = x0 + ah and y = y0 + bh. Using the Chain Rule, we find
f dx f dy
g 0 (h) = + = fx (x, y)a + fy (x, y)b.
x dh y dh
Letting h = 0 in the above expression, we find

f~u (x0 , y0 ) = g 0 (0) = fx (x0 , y0 )a + fy (x0 , y0 )b (11.6.1)

Example 11.6.1  
Find u~v (4, 0) if u(x, y) = x + y 2 and ~v = 21 , 23 .

Solution.
We have   !
1 3 1
u~v (4, 0) = ux (4, 0) + uy (4, 0) =
2 2 2

The Gradient Vector


The gradient vector is a generalization of the usual concept of derivative of a
function of one variable to functions of several variables. For a function u(x, y)
or u(x, y, z), the gradient vectors are, respectively,

u(x, y) = ux~i + uy~j and u(x, y, z) = ux~i + uy~j + uz~k.

Example 11.6.2
Let F (x, y, z) = u(x, y) z. Find F (x, y, z).

Solution.
We have
F (x, y, z) = ux~i + uy~j ~k

Example 11.6.3
Find the gradient vector of f (x, y, z) = (2x 3y + 5z)5 .

2
Solution.
We have

fx (x, y, z) =10(2x 3y + 5z)4


fy (x, y, z) = 15(2x 3y + 5z)4
fz (x, y, z) =25(2x 3y + 5z)4 .

Thus,
f (x, y, z) = 5(2x 3y + 5z)4 [2~i 3~j + 5~k]
With the notation for the gradient vector, we can rewrite the expression (11.6.1)
for the directional derivative as

f~u (x0 , y0 ) = f (x0 , y0 ) ~u.

This expresses the directional derivative in the direction of ~u as the scalar pro-
jection of the gradient vector onto ~u.

Maximizing the Directional Derivative


Suppose we have a function of two or three variables and we consider all possible
directional derivatives of f at a given point. These give the rates of change of
f in all possible directions. We can then ask the question: In which of these
directions does f achieves its maximum rate of change? The answer is provided
by the following theorem.

Theorem 11.6.2
The maximum value of the directional derivative of a function f (x, y) or f (x, y, z)
at a point (x, y) or (x, y, z) is ||f || and it occurs in the direction of the gradient
of f at that point.

Proof.
We have
f~u (x, y) = f ~u = ||f ||||~u|| cos = ||f || cos ,
where is the angle between f and ~u. The maximum value of cos is 1 and
this occurs when = 0. Therefore the maximum value of f~u is ||f || and it
occurs when = 0, that is, when ~u has the same direction as f

Example 11.6.4
Find the maximum rate of change of the function u(x, y) = 50 x2 2y 2 at the
point (1, 1).

Solution.
The maximum rate of change occurs in the direction of the gradient vector:

u(1, 1) = ux (1, 1)~i + uy (1, 1)~j = 2~i + 4~j.

The maximum rate of change at (1, 1) is


p
||u(1, 1)|| = (2)2 + 42 = 2 5

3
Tangent Plane to a Level Surface
Recall that a smooth curve in 3-D can be defined parametrically by the equations
x = x(t), y = y(t), z = z(t), where t is a parameter. This curve can be described
by the vector function

~r(t) = x(t)~i + y(t)~j + z(t)~k.

Its derivative is the tangent vector to the curve and is given by


d dx dy dz
(~r(t)) = ~i + ~j + ~k.
dt dt dt dt
Also, recall that the equation F (x, y, z) = C is called a level surface of
F (x, y, z). An important property of the gradient of F is that it is normal
to a level surface of F at every point. To see this, let S be the level sur-
face F (x, y, z) = k and P0 (x0 , y0 , z0 ) be an arbitrary point on S. Let C be
any curve on S that passes through P0 . We can describe C in parametric form
x = x(t), y = y(t), and z = z(t). Any point on C satisfies F (x(t), y(t), z(t)) = k.
Differentiating both sides of this equation with respect to t we find by means of
the Chain Rule

Fx (x, y, z)x0 (t) + Fy (x, y, z)y 0 (t) + Fz (x, y, z)z 0 (t) = 0

which can be written as F r0~(t) = 0. In particular, if x(t0 ) = x0 , y(t0 ) = y0


and z(t0 ) = z0 then
F (x0 , y0 , z0 ) r~0 (t0 ) = 0.
This means that the gradient vector F (x0 , y0 , z0 ) is perpendicular to any tan-
gent vector r~0 (t0 ) to any curve C on S that passes through P0 . See Figure 11.6.2.
Hence, the gradient vector is normal to the level surface at P0 .

Figure 11.6.2
The equation of the tangent plane to S at P0 is given by

Fx (x, y, z)(x x0 ) + Fy (x, y, z)(y y0 ) + Fz (x, y, z)(z z0 ) = 0.

4
Example 11.6.5
Find the equations of the tangent plane and normal line at the point (2, 1, 3)
to the ellipsoid
x2 z2
+ y2 + = 3.
4 9
Solution.
x2
The given ellipsoid is a level surface (k = 3) of the function f (x, y, z) = 4 +
2
y 2 + z9 . Since
x
fx (x, y, z) = , fx (2, 1, 3) = 1
2
fy (x, y, z) =2y, fy (2, 1, 3) =2
2 2
fz (x, y, z) = z, fz (2, 1, 3) = ,
9 3
the equation of the tangent plane is
2
(x + 2) + 2(y 1) (z + 3) = 0 = 3x 6y + 2z + 18 = 0.
3
The parametric equations of the normal line are
2
x(t) = 2 t, y(t) = 1 + 2t, z(t) = 3 t
3

5
Arkansas Tech University
MATH 2934: Calculus III
Dr. Marcel B Finan

11.7 Maximum and Minimum Values


Just like functions of a single variable, functions of several variables can have
local and global extrema, i.e., local and global maxima and minima.
We say that f (x, y) has a global maximum at a point (a, b) of its domain Df
if f (x, y) f (a, b) for all points (x, y) in Df . That is, f (a, b) is the largest value
of f in Df .
We say that f (x, y) has a global minimum at a point (a, b) of its domain Df
if f (a, b) f (x, y) for all points (x, y) in Df . That is, f (a, b) is the smallest
value of f in Df .
We say that f (x, y) has a local maximum at a point (a, b) of its domain Df if
there is an R > 0 such that f (x, y) f (a, b) for all points (x, y) in Df satisfying
(x a)2 + (y b)2 < R2 .
We say that f (x, y) has a local minimum at a point (a, b) of its domain Df if
there is an R > 0 such that f (a, b) f (x, y) for all points (x, y) in Df satisfying
(x a)2 + (y b)2 < R2 .
Collectively, local maxima and local minima are called local extrema. Similar
definition for global extrema. Figure 11.7.1 provides an example of a function
with local and global extrema.

Figure 11.7.1
Recall that for single-variable functions y = f (x), if x = c is a local maximum or
a minimum point, then either f 0 (c) = 0 or f 0 (c) does not exist. A point (c, f (c))
such f 0 (c) = 0 or f 0 (c) does not exist is called a critical point. Thus, the
recipe for finding a maximum or a minimum point is to locate critical points.
Something similar happens for functions of two variables.
Points where the gradient is either zero or undefined are called critical points
of a function f (x, y).
We next show that local extrema are critical points.

1
Theorem 11.7.1
If f has a local maximum or a local minimum at a non-boundary point (a, b) in
its domain then f (a, b) = ~0. That is, (a, b) is a critical point.

Proof.
Suppose f has a local extremum at a point (a, b). Define g(x) = f (x, b). Then
g(x) has a local extremum at x = a so that g 0 (a) = 0 = fx (a, b). Like-
wise, the function G(y) = f (a, y) has a local extremum at y = b so that
G0 (b) = 0 = fy (a, b). Hence, f (a, b) = ~0

Recall that the tangent plane to the surface z = f (x, y) is z = f (a, b) +


fx (a, b)(x a) + fy (a, b)(y b) so from the above theorem this equation be-
comes z = f (a, b). That is, the tangent plane at a local extremum is horizontal.
Just as the vanishing of the first derivative of a function in one variable does
not guarantee a maximum or a minimum, the vanishing of the gradient does
not guarantee a local extremum either. That is, the converse to Theorem 11.7.1
is not true in general.

Example 11.7.1
Locate and classify the critical points of f (x, y) = x2 y 2 .

Solution.
The gradient of f is given by f (x, y) = 2x~i 2y~j. We see that fx (x, y) = 2x
and fy (x, y) = 2y are simultaneously zero at (0, 0). Therefore, (0, 0) is a critical
point and a possible extremum. The graph of f (x, y) shown in Figure 11.7.2
indicates that (0, 0) is neither a local maximum nor a local minimum. Such a
point will be called a saddle point

Figure 11.7.2
We need to be able to determine whether or not a function has an extreme value
at a critical point. The following test is analogous to the Second Derivative Test

2
for functions of one variable.

The Second Derivative Test


Let (a, b) be a point in the domain of f such that fx (a, b) = fy (a, b) = 0.
Furthermore, let

fxx fxy
D(a, b) =
= fxx (a, b)fyy (a, b) [fxy (a, b)]2 .
fyx fyy

1. If D > 0 and fxx (a, b) > 0, then f (x, y) has a relative minimum at (a, b).
2. If D > 0 and fxx (a, b) < 0, then f (x, y) has a relative maximum at (a, b).
3. If D < 0 then f (x, y)) has a saddle point at (a, b).

Example 11.7.2
Find the local extrema and saddle points of the function

1 3 y2
f (x, y) = x 3x2 + + xy + 13x y + 2.
3 4
Solution.
we first find the critical points for this function. This gives us:

fx (x, y) =x2 6x + y + 13 = 0
y
fy (x, y) = + x 1 = 0.
2
From the second equation we find y = 2 2x. Substituting this into the first
equation we find x2 8x + 15 = (x 3)(x 5) = 0. Thus, x = 3 and x = 5 so
that the critical points are (3, 4) and (5, 8).
On the other hand, we have fxx (x, y) = 2x 6, fyy (x, y) = 21 , and fxy (x, y) = 1.
Hence, D(3, 4) = 1 < 0 so (3, 4) is a saddle point. Similarly, D(5, 8) =
2 1 = 1 > 0 and fxx (5, 8) = 4 > 0 so that (5, 8) is a local minimum

Example 11.7.3
Find the local extrema and saddle points of the function

f (x, y) = x3 + y 5 3x 10y + 4.

Solution.
The partial derivatives give

fx (x, y) =3x2 3 = 0
fy (x, y) =5y 4 10 = 0.

Solvingeach equation
we find
x = 1 and
y = 4 2. Thus, the critical points
4 4 4 4
are (1, 2), (1, 2), (1, 2), (1, 2).The discriminant is

D(x, y) = fxx (x, y)fyy (x, y) [fxy (x, y)]2 = 120xy 3 .

3

Since D(1, 4 2) = 120
4
8 and fxx (1, 4 2) = 6
> 0, (1, 4 2) is a local mini-
4 4
mum. Since D(1,
2) = 120 8 < 0, (1, 4 2) is a saddle point. Since

4 4 4
D(1,
2) = 120 8 < 0,(1, 2) is a saddle point.
Finally, since D(1, 4 2) =
120 4 8 > 0 and fxx (1, 4 2) = 6 < 0, (1, 4 2) is a local maximum

The second derivative test discussed above, did not cover the case D = 0.
As illustrated in the example below, the second derivative test is inconclusive
in this case. That is one cannot classify the critical point. It can be either a
local maximum, a local minimum or a saddle point.

Example 11.7.4
Let f (x, y) = x4 + y 4 , g(x, y) = x4 y 4 , and h(x, y) = x4 y 4 . Show that
D(0, 0) = 0 for each function. Classify the critical point (0, 0) for each function.

Solution.
Note that fx (0, 0) = fy (0, 0) = 0 so that f (x, y) has a critical point at (0, 0).
Since fxx (x, y) = 12x2 , fyy (x, y) = 12y 2 and fxy (x, y) = 0, we have D(0, 0) =
fxx (0, 0)fyy (0, 0) [fxy (0, 0)]2 = 0. But the smallest value of f (x, y) occurs at
(0, 0) so that f (x, y) has a local and global minimum at (0, 0) with D(0, 0) = 0.
Similarly, gx (0, 0) = gy (0, 0) = 0 so that (0, 0) is a critical point of g. Moreover,
gxx (x, y) = 12x2 , gyy (x, y) = 12y 2 and gxy (x, y) = 0, we have D(0, 0) =
gxx (0, 0)gyy (0, 0) [gxy (0, 0)]2 = 0. Since g(x, y) 0, the largest value occurs
at (0, 0). That is, g has a local and global maximum at (0, 0) with D(0, 0) = 0.
Finally, we have hx (0, 0) = hy (0, 0) = 0 so that (0, 0) is a critical point of h.
Since hxx (x, y) = 12x2 , hyy (x, y) = 12y 2 and hxy (x, y) = 0, we have D(0, 0) =
hxx (0, 0)hyy (0, 0) [hxy (0, 0)]0 = 0. However, h(0, 0) = 0, z = h(x, 0) = x4 > 0
and z = h(0, y) = y 4 < 0. Hence, (0, 0) is a saddle point with D(0, 0) = 0

Example 11.7.5
Find the shortest distance from the point (1, 0, 2) to the plane x + 2y + z = 4.

Solution.
Let d be the distance from (1, 0, 2) to any point (x, y, z) on the plane x + 2y +
z = 4. By the distance formula,
p p
d = (x 1)2 + y 2 + (z + 2)2 = (x 1)2 + y 2 + (6 x 2y)2 .

We have
2x + 2y 7
fx (x, y) =
d
2x + 5y 12
fy (x, y) = .
d
Solving 2x + 2y 7 = 0 and x + 5y 12 = 0 simultaneously gives x = 11 6 and
y = 53 so that ( 11 ,
6 3
5
) is the only critical point of f. An absolute minimum exists
(since there is a minimum distance from the point to the plane) and it must
occur at a critical point so the shortest distance occurs when x = 11 5
6 and y = 3 ,

4
for which d = 5
6

Absolute Extrema
In real life, one is most likely interested in finding the places at which the largest
and smallest values of a function f occur in its domain.
We recall the reader that a point (a, b) in the domain of f (x, y) is called an
absolute or global maximum if f (x, y) f (a, b) for all points in the domain
of f.
If f (a, b) f (x, y) for all points in the domain of f then f (x, y) has an abso-
lute or global minimun at (a, b).
Optimization typically refers to finding the global maximum or minimum of
a function. If the domain of f is the entire xyplane then we have an uncon-
strained optimization; if the domain of f is not the entire xyplane then we
have a constrained optimization.

Example 11.7.6 (Unconstrained Optimization)


Consider the function f (x, y) = x2 (y + 1)3 + y 2 . Find the global extrema of f,
if they exist.

Solution.
The first partials give

fx (x, y) =2x(y + 1)3 = 0


fy (x, y) =3x2 (y + 1)2 + 2y = 0.

This implies that the only critical point is (0, 0). Finding second partials we
have

fxx (x, y) =2(y + 1)3 fxx (0, 0) =2


2
fyy (x, y) =6x (y + 1) + 2 fyy (0, 0) =2
2
fxy (x, y) =6x(y + 1) fxy (0, 0) =0.

Since D = fxx (0, 0)fyy (0, 0) fxy (0, 0)2 = 4 > 0 and fxx (0, 0) = 2 > 0, the
point (0, 0) is a local minimum. Since f (3, 2) = 5 < f (0, 0) = 0 the point
(0, 0)is not a global minimum. Thus, f has no global extrema

Like functions in one variable, a function f (x, y) can have both a global maxi-
mum and a global minimum; a global maximum but no global minima; a global
minimum but no global maxima; or none. So are there conditions that guar-
antee that a function has a global maximum and global minimum? In single
variable calculus we saw that a function f (x) continuous on a closed (i.e., in-
cluding the endpoints) and bounded (i.e. of finite length) interval has both a
global maximum and a global minimum. A similar result is true for functions
of two variables. However, we need to define what we mean by bounded and
closed in 2D case.

5
A closed set in IR2 is one which contains its boundary and with no holes in its
interior. For example, the disk x2 + y 2 1 is a closed set whereas x2 + y 2 < 1
is not since the boundary, which is the circumference of the circle x2 + y 2 = 1,
is not included. Similarly, 0 < x2 + y 2 1 is not closed since it has a hole
at the origin. A bounded set in IR2 is one that can be contained in a disk
(x a)2 + (y b)2 < R.
Using these definitions, we have the following theorem for multivariable func-
tions:

Theorem 11.7.2 ( Extreme Value Theorem for Multivariable Functions)


If f is a continuous function on a closed and bounded set D in IR2 then f has
a global maximum and a global minimum in D.

We note that if f is not continuous or the set D is not closed or bounded, then
there is no guarantee that f will have a global maximum or minimum. For
example, the plane f (x, y) = x + y 1 is continuous in the entire plane but does
not have global extrema since the set is not bounded.
Just as in the case of single variable functions, one can find the global extrema
by doing the following:
Step 1. Find the values of f at the critical points of f in D.
Step 2. Find the extreme values of f on the boundary of D.
Step 3. The largest of the values from steps 1 and 2 is the absolute maximum
value; the smallest of these values is the absolute minimum value.

Example 11.7.7
Find the absolute extrema of the function f (x, y) = x + y xy in the closed
triangle with vertices at (0, 0), (0, 2) and (4, 0).

Solution.
Since D is closed and bounded and f is continuous on D, Theorem 11.7.2 guar-
antees that f has global extrema in D.
Step 1. The critical points are solutions to the equations fx (x, y) = 0 and
fy (x, y) = 0. That is, 1 y = 0 and 1 x = 0. The only critical point is (1, 1)
and f (1, 1) = 1.
Step 2. Along the line from (0, 0) to (0, 2), the function f (0, y) = y has a maxi-
mum value of 2 at (0, 2) and a minimum value of 0 at (0, 0). Along the line from
(0, 0) to (4, 0) the function f (x, 0) = x has a maximum value of 4 at (4, 0) and
a minimum of 0 at (0, 0). Along the line from (4, 0) to (0, 2), i.e., y = 2 x2 , we
2
have f (x, y) = f (x, 2 x2 ) = 12 x 32 + 78 for 0 x 4, a quadratic function
with minimum at x = 32 , where f ( 32 , 54 ) = 87 and a maximum at x = 4, where
f (4, 0) = 4.
Step 3. The absolute maximum of f on D is f (4, 0) = 4 and the absolute
minimum is f (0, 0) = 0

6
Arkansas Tech University
MATH 2934: Calculus III
Dr. Marcel B Finan

11.8 Constraint Optimization: Lagranges Multipliers


Most optimization problems encountered in the real world are constrained by
external circumstances. For example, a city wanting to improve its roads has
only limited number of tax dollars it can spend on the project.
Constrained optimization is the maximization or minimization of an objec-
tive function subject to constraints on the possible values of the independent
variable(s). Constraints can be either equality constraints or inequality con-
straints. In this section, we see how to find an optimum value of a function
of two variables subject to some constraints using a graphical approach and an
analytical approach that employs the so-called Lagranges Multipliers.

Graphical Approach
We consider the following example. A company has determined that its pro-
2 1
duction function (in units) is the Cobb-Douglas function f (x, y) = x 3 y 3 where
x and y are raw materials. If x and y cost $1000 per unit and $3780 is the
budget available , then what is the maximum production that can be obtained
that exhausts the available budget?
Mathematically, we are asked to maximize the function f subject to the con-
straint g(x, y) = 3.78 where g(x, y) = x + y. Graphically, the line x + y = 3.78
represents all the combinations of raw materials that just exhaust the budget
but are still affordable.
The maximum production can be located graphically by plotting contours of f
on a plot containing the line x + y = 3.78 as shown in Figure 11.8.1.
To maximize f we find the point which lies on the level curve with the largest
possible value of f and which lies on the line x + y = 3.78. This figure shows
that at the maximum, f must be tangent to the constraint line. Note that the
maximum value of f is about 2 and this occurs at x 2.52 and y 1.26.
In general, the global maximum or global minimum occurs where the graph
of the constraint equation is tangent to one of the level curves of the original
function.

Figure 11.8.1

1
Analytical Approach: Lagrange Multipliers
As noted above, the maximum production is achieved at the point where the
constraint is tangent to a level curve of the production function. The method of
Lagrange Multipliers uses this fact in algebraic form to calculate the maximum.
From Figure 11.8.1, we see that at the point of tangency, f (x, y) and g(x, y)
are parallel so that f (x, y) = g(x, y) for some which we call the Lagrange
multiplier. We therefore have
s 
r 2
2 3 y~ 1 3 x ~
i+ j = ~i + ~j.
3 x 3 y

Hence,
r 
2
2 3 y 1 3 x
p
3 x = and 3 y = .

Eliminating gives
r 
2
2 3 y 1 x
p 3
3 x = 3 y which leads to 2y = x.

Substituting this into the equation x + y = 3.78 we find x = 2.52 and y = 1.26.
Hence,
2 1
f (2.52, 1.26) = (2.52) 3 (1.26) 3 2.
As before, we see that the maximum value of f is approximately 2. Also, note
that 0.53

Generalization
We are given a constraint equation

g(x, y) = c

and an objective function f (x, y). The goal is to find the global maximum and
minimum values of f among the values taken on by f along the constraint curve;
i.e., the set of points for which g(x, y) = c. Moreover, we would like to find all
of the points (x, y) at which these maxima and minima are attained. This is
provided by the following theorem.

Theorem 11.8.1
Suppose that g(x, y) 6= ~0 on the curve g(x, y) = c. If there is a maximum or
a minimum of the values that the function f (x, y) assumes on the constraint
curve g(x, y) = c, then it occurs at a point (x, y) at which

f (x, y) = g(x, y) and g(x, y) = c.


Compute the values of f at these points. The largest value is the global maxi-
mum and the smallest value is the global minimum.

2
Example 11.8.1
Find the maximum and minimum of f (x, y) = 5x 3y subject to the constraint
x2 + y 2 = 136.

Solution.
Since f (x, y) = 5~i 3~j and g(x, y) = 2x~i + 2y~j, we must have 2x = 5
5 3
and 2y = 3. Eliminating we find 2x = 2y and this leads to y = 53 x.
9 2
Substituting into the constraint equation we find x2 + 25 x = 136. Solving
this equation for x we find x = 10. If x = 10 then y = 6 and if x = 10
then y = 6. Note that g 6= ~0 on the circle. Since, f (10, 6) = 68 and
f (10, 6) = 68, the maximum of f occurs at the point (10, 6) and the mini-
mum occurs at the point (10, 6)

Optimization with Closed and Bounded Constraints


If the constraint is a closed and bounded region then by the Extreme Value
Theorem of the previous section, the global maximum or minimum occurs ei-
ther at the critical points of f inside the region or at a point on the boundary
of the region. Lets work an example to see how these kinds of problems work.

Example 11.8.2
Find the maximum and minimum values of f (x, y) = 4x2 + 10y 2 on the disk
x2 + y 2 4.

Solution.
The first step is to find all the critical points that are inside the disk (i.e., satisfy
the constraint x2 + y 2 < 4.). We have

fx (x, y) =8x = 0
fy (x, y) =20y = 0

So (0, 0) is the only critical point of f satisfying x2 + y 2 < 4.


Next, we proceed with Lagrange Multipliers and we treat the constraint as an
equality instead of the inequality. Since f (x, y) = 8x~i + 20y~j and g(x, y) =
2x~i + 2y~j, we must have 2x = 8x and 2y = 20y. These equations imply

2x( 4) =0
2y( 10) =0.

If x = 0 we find y = 2. If x 6= 0 then = 4 and so the second equation gives


y = 0 and so x = 2. If we had performed a similar analysis on the second
equation we would arrive at the same points.
So, Lagrange Multipliers gives us four boundary points to check : (0, 2), (0, 2), (2, 0), (2, 0).
Now, since

f (0, 0) =0
f (2, 0) =f (2, 0) = 16
f (0, 2) =f (0, 2) = 40

3
the global maximum of f occur at the points (0, 2) and (0, 2) and the global
minimum occurs at (0, 0)

Practical Interpretation of
Let (x , y ) be an optimum value. Then its location depends on c where
g(x, y) = c. Thus, x = x (c) and y = y (c). Moreover, we can look to f
as a composite function f (x (c), y (c)). Using the chain rule we can write

df f dx f dy
= + .
dc x dc y dc
However, we have
f g f g
x (x , y ) = x (x , y ) and
y (x , y ) = y (x , y )

so that
df g dx g dy dg
= + = .
dc x dc y dc dc
dg
Since g(x (c), y (c)) = c we must have dc = 1. Thus,

df
= .
dc
This says that is the rate of change of the optimum value of f as c increases.
For example, in the budget function discussed earlier, an increase of $1000 in
the budget will lead to an increase of about 0.53 unit in production.

4
Arkansas Tech University
MATH 2934: Calculus III
Dr. Marcel B Finan

12.1 The Definite Integral of f (x, y)


In this section, we introduce the concept of definite integral of a function of two
variables over a rectangular region.
By a rectangular region we mean a region R as shown in Figure 12.1.1(I).

Figure 12.1.1
Let f (x, y) be a continuous function on R. Our definition of the definite integral
of f over the rectangle R will follow the definition from one variable calculus.
Partition the interval a x b into n equal subintervals using the mesh points
a x0 < x1 < x2 < < xn1 < xn = b with xi = ba n denoting the length
of each subinterval. Similarly, partition c y d into m subintervals using the
mesh points c = y0 < y1 < y2 < < ym1 < ym = d with y = dc m denoting
the length of each subinterval. This way, the rectangle R is partitioned into mn
subrectangles each of area equals to xy as shown in Figure 12.1.2(II).
Let Dij be a typical rectangle. Pick a point (xi , yj ) in this rectangle. The sum
m X
X n
f (xi , yj )xy
j=1 i=1

is called a Riemann sum of f (x, y) over R.


We define the double integral of f over the rectangle R to be
Z Z X n
m X
f (x, y)dxdy = lim f (xi , yj )xy
R m,n
j=1 i=1

provided that the limit exists.

Double Integral as Volume Under a Surface


Just as the definite integral of a positive one-variable function can be inter-
preted as area, so the double integral of a positive two-variable function can be
interpreted as a volume.
Let f (x, y) > 0 with surface S shown in Figure 12.1.2(I). Partition the rectangle
R as above. Over each rectangle Dij we will construct a box whose height is
given by f (xi , yj ) as shown in Figure 12.1.2 (II). Each of the boxes has a base
area of xy and a height of f (xi , yj ) so the volume of each of these boxes is
f (xi , yj )xy. So the volume under the surface S is then approximately,
m X
X n
V f (xi , yj )xy.
j=1 i=1

1
As the number of subdivisions grows, the tops of the boxes approximate the
surface better, and the volume of the boxes gets closer and closer to the volume
under the graph of the function. Thus, we have the following result:
If f (x, y) > 0 then the volume under the graph of f above the region R is
Z Z
f (x, y)dxdy.
R

Figure 12.1.2

Remark 12.1.1
If we choose a function f (x, y) = 1 everywhere in R then our integral becomes:
RR
Area of R = R
1dxdy.
That is, when f (x, y) = 1, the integral gives us the area of the region we are
integrating over.

Example 12.1.1
Use the Riemann sum with n = 3, m = 2 and sample point the upper right
corner of each subrectangle to estimate the volume under z = xy and above the
rectangle 0 x 6, 0 y 4.

Solution.
The interval on the xaxis is to be divided into n = 3 subintervals of equal
length, so x = 603 = 2. Likewise, the interval on the yaxis is to be divided
into m = 2 subintervals, also with width y = 2; and the rectangle is divided
into six squares with sides 2.
Next, the upper right corners are at (2, 2), (4, 2) and (6, 2), for the lower
three squares, and (2, 4), (4, 4) and (6, 4), for the upper three squares. The
approximation is then

[f (2, 2) + f (4, 2) + f (6, 2))+f (2, 4) + f (4, 4) + f (6, 4)] 2 2


=[4 + 8 + 12 + 8 + 16 + 24] 4 = 72 4 = 288

2
The Average of f (x, y)
As in the case of single variable calculus, the average value of f (x, y) over a
region R is defined by
Z Z
1
f (x, y)dxdy.
Area of R R

Example 12.1.2
Estimate the average value of f (x, y) in Example 12.1.1.

Solution.
We have Z Z
1 288
f (x, y)dxdy = 12
Area of R R (6)(4)

Iterated Integrals
As seen above, we used Riemann sums to approximate a double integral. Next,
we see how to compute double integrals exactly using one-variable integrals.
Going back to our definition of the integral over a region as the limit of a double
Riemann sum:
Z Z Xm X n
f (x, y)dxdy = lim f (xi , yj )xy
R m,n
j=1 i=1
m n
!
X X
= lim f (xi , yj )x y
m,n
j=1 i=1
m n
!
X X
= lim y f (xi , yj )x
m,n
j=1 i=1
m
X Z b
= lim y f (x, yj )dx.
m a
j=1

We now let Z b
F (yj ) = f (x, yj )dx
a
and, substituting into the expression above, we obtain
Z Z m
X Z d Z d Z b
f (x, y)dxdy = lim F (yj )y = F (y)dy = f (x, y)dxdy.
R m c c a
j=1

Thus, if f is continuous over a rectangle R then the integral of f over R can


be expressed as an iterated integral. To evaluate this iterated integral, first
perform the inside integral with respect to x, holding y constant, then integrate
the result with respect to y.

3
ExampleR12.1.3
16 R 8 x y

Compute 0 0 12 4 8 dxdy.

Solution.
We have
Z 16 Z 8 Z 16 Z 8 
 x y x y 
12 dxdy = 12 dx dy
0 0 4 8 0 0 4 8
Z 16  8
x2 xy
= 12x dy
0 8 8 0
Z 16 16
y 2
= (88 y)dy = 88y = 1280
0 2 0

We note, that we can repeat the argument above for establishing the iterated
integral, reversing the order of the summation so that we sum over j first and
i second (i.e., integrate over y first and x second) so the result has the order of
integration reversed. That is, we can show that
Z Z Z b Z d
f (x, y)dxdy = f (x, y)dydx.
R a c

ExampleR12.1.4
8 R 16 x y

Compute 0 0 12 4 8 dydx.

Solution.
We have
Z 8 Z 16  Z 8 Z 16 
x y  x y
12 dydx = 12 dy dx
0 0 4 8 0 0 4 8
Z 8 2 16

xy y
= 12y dx
0 4 16 0
Z 8
8
= (176 4x)dx = 176x 2x2 0 = 1280
0

Properties of Double Integrals


The following properties hold for double integrals:
Z Z Z Z Z Z
[f (x, y) g(x, y)]dxdy = f (x, y)dxdy g(x, y)dxdy
R R R
Z Z Z Z
cf (x, y)dxdy = c f (x, y)dxdy
R R
Z Z Z Z
f (x, y)dxdy g(x, y)dxdy, if f (x, y) g(x, y) for all (x, y) in R.
R R

4
Arkansas Tech University
MATH 2934: Calculus III
Dr. Marcel B Finan

12.2 Double Integrals Over General Bounded Regions


In Section 12.1, we looked at double integrals over rectangular regions. The
problem with this is that most of the regions are not rectangular so we need to
now look at the following double integral,
Z Z
f (x, y)dxdy
R

where R is any bounded region. In this case, we let D be the smallest rectangle
containing R and we define the function F (x, y) = f (x, y) in R and F (x, y) = 0
for (x, y) in D but not in R. We define
Z Z Z Z
f (x, y)dA = F (x, y)dA.
R D

We consider the two types of regions R shown in Figure 12.2.1.

Figure 12.2.1

In Type 1, we let D = [a, b][c, d] be the smallest rectangle containing R. Then,


we have
Z Z Z Z Z bZ d
f (x, y)dA = F (x, y)dA = F (x, y)dydx.
R D a c

But F (x, y) = 0 for y < g1 (x) and y > g2 (x) because (x, y) then lies outside R.
Hence,
Z d Z g2 (x)
F (x, y)dy = f (x, y)dy
c g1 (x)

1
and the iterated integral of f over R is defined by
Z Z Z b Z g2 (x)
f (x, y)dxdy = f (x, y)dydx.
R a g1 (x)

This means, that we are integrating using vertical strips from g1 (x) to g2 (x)
and moving these strips from x = a to x = b.
In Type 2, we have
Z Z Z d Z h2 (y)
f (x, y)dxdy = f (x, y)dxdy
R c h1 (y)

so we use horizontal strips from h1 (y) to h2 (y). Note that in both cases, the
limits on the outer integral must always be constants.

Remark 12.2.1
Choosing the order of integration will depend on the problem and is usually
determined by the function being integrated and the shape of the region R. The
order of integration which results in the simplest evaluation of the integrals
is the one that is preferred.

Example 12.2.1
Let f (x, y) = xy. Integrate f (x, y) for the triangular region bounded by the
xaxis, the yaxis, and the line y = 2 2x.

Solution.
Figure 12.2.2 shows the region of integration for this example.

Figure 12.2.2
Graphically, integrating over y first is equivalent to moving along the x axis
from 0 to 1 and integrating from y = 0 to y = 2 2x. That is, summing up the

2
vertical strips as shown in Figure 12.2.3(I).
Z Z Z 1 Z 22x
xydxdy = xydydx
R 0 0
1 22x
xy 2 1 1
Z Z
= dx = x(2 2x)2 dx
0 2 0 2 0
Z 1  2 1
2 3 x 2 3 x4 1
=2 (x 2x + x )dx = 2 x + = .
0 2 3 4 0 6

If we choose to do the integral in the opposite order, then we need to invert


y = 2 2x, i.e. express x as function of y. In this case we get x = 1 12 y.
Integrating in this order corresponds to integrating from y = 0 to y = 2 along
horizontal strips ranging from x = 0 to x = 1 12 y, as shown in Figure 12.3.3(II)
Z Z Z 2 Z 1 12 y
xydxdy = xydxdy
R 0 0
1 1 y
2
x2 y 2 1 2
Z Z
1
= dy = y(1 y)2 dy
0 2
0 2 0 2
Z 2 3 2 3
2
1 2 y y y y 4 1
= (y y + )dy = + =
2 0 4 4 6 32 0 6

Figure 12.3.3

Example
R R 12.2.2 3
Find R
(4xy y )dxdy where R is the region bounded by the curves y = x
and y = x3 .

3
Solution.
A sketch of R is given in Figure 12.2.4. Using horizontal strips we can write
Z Z Z 1Z 3 y
3
(4xy y )dxdy = (4xy y 3 )dxdy
R 0 y2
Z 1 3 y
Z 1  
5 10
2
= 2x y xy 3 y2 dy = 2y 3 y 3 y 5 dy
0 0
1
3 8 3 13 1 55
= y 3 y 3 y 6 =
4 13 6 0 156

Figure 12.2.4

Example 12.2.3
R 2 R 4x2
Sketch the region of integration of 0 4x2 xydydx

Solution.
A sketch of the region is given in Figure 12.2.5

Figure 12.2.5

4
Properties of Double Integrals
TheR following
R properties of double integrals
RR hold: RR
(i) R
[f (x, y) + g(x, y)]dA = R
f (x, y)dA
R R + R
g(x, y)dA.
RR
(ii) If f (x, y) g(x, y) for
RR all (x, y) in R, then R
f (x, y)dA R
g(x, y)dA.
(iii) If f (x, y) = 1 then R
1dA is the volume of the cylinder with base R and
height 1 which is equal to the area of R.
(iv) If m f (x, y) M for all (x, y) in R and A(R) is the area of R, then
Z Z
mA(R) f (x, y)dA M A(R).
R

5
Arkansas Tech University
MATH 2934: Calculus III
Dr. Marcel B Finan

12.3 Double Integrals in Polar Coordinates


There are regions in the plane that are not easily used as domains of iterated
integrals in rectangular coordinates. For instance, regions such as a disk, ring,
or a portion of a disk or ring.
We start by recalling the relationship between Cartesian and polar coordinates.
The Cartesian system consists of two rectangular axes. A point P in this system
is uniquely determined by two numbers x and y as shown in Figure 12.3.1(a).
The polar coordinate system consists of a point O, called the pole, and a half-
axis starting at O and pointing to the right, known as the polar axis. A point
P in this system is determined by two numbers: the distance r between P and
O and an angle between the ray OP and the polar axis as shown in Figure
12.3.1(b).

Figure 12.3.1
The Cartesian and polar coordinates can be combined into one figure as shown
in Figure 12.3.2.
Figure 12.3.2 reveals the relationship between the Cartesian and polar coordi-
nates:
y = r sin tan = xy .
p
r = x2 + y 2 x = r cos

Figure 12.3.2

1
A double integral in polar coordinates can be defined as follows. Suppose we
have a region
R = {(r, ) : a r b, }
as shown in Figure 12.3.3(a).

Figure 12.3.3
Partition the interval into m equal subintervals, and the interval
a r b into n equal subintervals, thus obtaining mn subrectangles as shown
in Figure 12.3.3(b). Choose a sample point (ri , j ) in the subrectangle Rij
defined by ri1 r ri and j1 j . Then
Z Z m X
X n
f (x, y)dxdy f (ri , j )Aij
R j=1 i=1

where Aij is the area of the subrectangle Rij .


To calculate the area of Rij , look at Figure 12.3.4. If r and are small then
Rij is approximately a rectangle with area ri r so

Rij ri r.

Thus, the double integral can be approximated by a Riemann sum


Z Z m X
X n
f (x, y)dxdy f (ri , j )ri r
R j=1 i=1

Taking the limit as m, n we obtain


Z Z Z Z b
f (x, y)dxdy = f (r, )rdrd.
R a

2
Figure 12.3.4

ExampleR 12.3.1
R x2 +y2
Evaluate R
e dxdy where R : x2 + y 2 1.

Solution.
We have

Z Z Z 1 Z 1x2
x2 +y 2 2
+y 2
e dxdy = ex dydx
R 1 1x2
Z 2 Z 1 Z 2  Z 1 
r2 2
= e rdrd = d er rdr
0 0 0 0
 1
2 1 r2
= []0 e
2 0
=(e 1)

Example 12.3.2
Compute the area enclosed by the unit circle.

Solution.
The area is given by

Z 1 Z 1x2 Z 2 Z 1

dydx = rdrd
1 1x2 0 0
2 1 1
r2
Z  Z  
2
= d rdr = []0
0 0 2 0
=

Example 12.3.3
1
Evaluate the double integral of f (x, y) = x2 +y 2 over the region D shown in

3
Figure 12.3.5.

Figure 12.3.5

Solution.
We have

! Z
Z Z 2 Z 2 
4 1 4 1
rdrd = d rdr
0 1 r2 0 1 r2
/4 2
= []0 [ln r]1

= ln 2
4
Example 12.3.4
For each of the regions shown in Figure 12.3.6, decide whether to integrate using
rectangular or polar coordinates. In each case write an iterated integral of an
arbitrary function f (x, y) over the region.

Figure 12.3.6

Solution.
R 2 R 3
(a) 0 0 f (r, )rdrd

4
R3R2
(b) f (x, y)dydx
R 12 R 1
3
(c) 0 1 f (x, y)dydx
2 x1

What we have done so far can be extended to the more complicated type of
region as shown in Figure 12.3.7.

Figure 12.3.7

In an argument similar to the one for type I region of Section 12.2, one can show
that Z Z Z Z h2 ()
f (x, y)dA = f (r cos , r sin )rdrd.
R h1 ()

Example 12.3.5
Find the area enclosed by one loop of the rose r = cos (3).

Solution.
One loop is given by the region R = {(r, ) : 6
6, 0 r cos (3)} as
shown in the Figure 12.3.8.

Figure 12.3.8

5
We have
Z Z Z /6 Z cos (3)
A= dA = rdrd
R /6 0
/6 r=cos (3)
r2
Z 
= d
/6 2 r=0
Z /6 Z /6    /6
1 1 1 + cos (6) 1 1
= cos2 (3)d = 2 d = + sin (6) =
/6 2 0 2 2 2 2 6 0 12

You might also like